Frage:
Ich glaube, ich habe ein berühmtes offenes Problem gelöst. Wie kann ich Leute auf dem Gebiet davon überzeugen, dass ich keine Kurbel bin?
user3439590
2014-03-25 04:51:49 UTC
view on stackexchange narkive permalink

Ich interessiere mich für die Situation, in der Sie ein sehr interessantes Ergebnis erzielen. Zum Beispiel haben Sie ein sehr wichtiges offenes Problem gelöst. Sie sind jedoch auf diesem Gebiet nicht bekannt und haben keine bemerkenswerten Veröffentlichungen. Ihr Vorgesetzter findet die Arbeit gut und Sie reichen die Arbeit in einem hochkarätigen Journal ein, werden jedoch abgelehnt.

Der Beitrag ist sehr stark. Es bricht, was die meisten Menschen glauben oder was sie bereits bewiesen haben: Sie lösen beispielsweise das P vs. NP-Problem oder ein anderes bekanntes offenes Problem.

Die Prüfer lehnen Ihre Arbeit ohne Begründung nachdrücklich ab und geben nicht an, warum das Ergebnis falsch ist. Beispiele für Überprüfungskommentare sind:

  • "Der Beweis muss falsch sein."
  • "Sie können ein solches Ergebnis nicht erzielen."
  • "Sie verstehen den Begriff ..."

Meine Frage ist, was in dieser Situation zu tun ist. Wo hin? Was sollten Sie tun, wenn Ihr Berater die Arbeit akzeptiert, die Prüfer des Top-Journals die Arbeit jedoch ablehnen, ohne die Fehler zu erklären?

In diesem Fall ist der Aufsichtsperson * wahrscheinlich * falsch in der Wahrnehmung seiner eigenen Arbeit; Erkennen, wofür Peer Review gedacht ist. Aber vielleicht auch nicht; Vielleicht ist die Präsentation nur schlecht oder die Behauptung für manche Herzen / Köpfe zu empörend. Laden Sie den Zeitstempel auf arXiv hoch und verbessern Sie das Formular und das Senden. Ihr Name spielt beim Einreichen eines Artikels keine Rolle (lesen: sollte nicht), sodass Unbekanntheit kein Problem ist (lesen: sollte nicht sein). Bekannt für halbgebackenes Kurbelzeug ist andererseits: Vermeiden Sie diesen Eindruck um jeden Preis!
Siehe auch [hier] (http://blog.computationalcomplexity.org/2009/01/so-you-think-you-settled-p-verus-np.html) und [hier] (http: //www.scottaaronson) .com / blog /? p = 304) für weitere Ratschläge. Und denken Sie daran, dass Nobelpreise und jahrhundertelanger Ruhm an Menschen gingen, die niemand in seiner (Lebens-) Zeit ernst nahm.
Ich sollte hinzufügen, dass der "Rat", den ich verlinke, offensichtlich ziemlich eigensinnig ist und mit einem Körnchen Salz genommen werden sollte. Nach allem, was wir wissen, haben Sie die Lösung für ein wichtiges offenes Problem. * Aber * Sie müssen die Situation berücksichtigen und Ihren Versuch entsprechend präsentieren, wenn Sie möchten, dass die Leute ihn ernst nehmen. Die Blog-Beiträge, die ich verlinke, sollen Ihnen einen Eindruck davon geben, wie empfindlich Domain-Experten sein können, wenn sie mit (für sie) offensichtlich falschen Versuchen bombardiert wurden, deren Autoren "Nein" nicht akzeptieren - seit * Jahrzehnten *. Schreiben Sie für * sie *.
Sie sollten diese Seite lesen: http: //research.microsoft.com/en-us/um/people/cohn/Thoughts/advice.html
1) Stellen Sie sicher, dass Ihre Ergebnisse tatsächlich korrekt sind, indem Sie sich an andere Pioniere auf dem Gebiet wenden (bitten Sie Ihren Vorgesetzten, dies zu tun). 2) Veröffentlichen Sie sie in Arxiv. 3) Warten Sie auf die Seminareinladungen und die Welt des Ruhms
@Raphael: Ich möchte Sie daran erinnern, dass der Nobelpreis bisher nur an lebende Personen vergeben wurde. Ich werde auch darauf hinweisen, dass nur weil sie alle über Einstein gelacht haben, das nicht bedeutet, dass Sie ein neuer Einstein sind, wenn sie über Sie lachen. Gute Links - danke dafür.
"Sie haben über Columbus gelacht, sie haben über Fulton gelacht, sie haben über die Gebrüder Wright gelacht. Aber sie haben auch über Bozo den Clown gelacht." -Carl Sagan (http://www.c2.com/cgi/wiki?TheyLaughedAtEinstein)
@EricLippert: [Nobel:] Das ist mir bewusst; Ich dachte an einige Künstler (und Ruhm, nicht an Nobelpreise), als ich das "(Leben)" dort platzierte. [Argumentumkehrung:] Offensichtlich. Die Tatsache, dass falsche Negative * passieren *, ist wichtig zu bedenken, wenn nur so ein angemessenes Maß an allgemeiner Knappheit auf diejenigen ausgedehnt wird, die es versuchen (und zu scheitern scheinen).
@Raphael ["Erkennen, wofür Peer Review gedacht ist."] (Http://www.theguardian.com/technology/shortcuts/2014/feb/26/how-computer-generated-fake-papers-flooding-academia)
@JoelReyesNoche, Natürlich haben die Leute in Bezug auf Mr. Sagan nicht über Columbus gelacht, weil er dachte, die Erde sei rund, sondern weil sie wussten, dass er den Durchmesser der Erde drastisch unterschätzt hatte (was zu einer geschätzten Reise von 3.700 km im Vergleich zu was führte wäre eine tatsächliche 19.600 km lange Reise gewesen). Columbus wurde nur finanziert, weil er Königin Isabella I und König Ferdinand II den Handel mit Asien versprach, zu einer Zeit, als das Paar dringend die Einnahmen aus dem Handel brauchte.
@Jigg "Der Beweis muss falsch sein." "Sie können ein solches Ergebnis nicht erzielen." "Sie verstehen den Begriff ... nicht gut."
Wenn Sie nur glauben, dass Sie es gelöst haben, sollten Sie sich zunächst selbst überzeugen, bevor Sie andere überzeugen. Erst wenn Sie alle Beweise und Details so klar aufgeschrieben haben, dass für Sie absolut kein Zweifel besteht und Sie WISSEN, dass Sie bewiesen haben, dass Sie sich Sorgen machen sollten, andere Menschen zu überzeugen;) .. Und behalten Sie im Hinterkopf, dass im Grunde jeder Der Forscher glaubte irgendwann in seiner Karriere, dass er ein Lemma / einen Satz / einen Satz bewiesen hatte, um später zu erkennen, dass er einen Fehler gemacht hatte ...
Eine [Kurbel] (http://en.wikipedia.org/wiki/Crank_ (Person)) wird von Wikipedia als jemand definiert, der nicht erkennen kann, dass er falsch liegt, selbst wenn Beweise für das Gegenteil vorgelegt werden. Es besteht keine Notwendigkeit, etwas zu tun, wenn Sie die Schlussfolgerung der Prüfer akzeptieren. Wenn Sie dies nicht tun, fragen Sie nach Gutachtern, die auf den Fehler hinweisen können? und _Können Sie akzeptieren, dass Sie sich irren und gegenteilige Beweise akzeptieren? _ Stackexchange hat Chat-Foren mit einigen Experten und schlägt in Ihrem Fall vor [Sie erwähnen die Informatik nicht, aber es ist in Ihrem Profil], [cs.se] a zu geben Schuss.
@Raphael Bin ich falsch, dass Sie nichts auf arxiv hochladen können, wenn Sie keine Akkreditierung für eine Universität / ein Forschungszentrum haben (wenn Sie beispielsweise in der Branche arbeiten)?
Das Grundproblem hierbei ist, dass Sie eine unbegründete Behauptung aufgestellt haben: Sie sind keine Kurbel. Ja, es gibt einige Fälle (z. B. Helicobacter), in denen es viel Zeit und Mühe gekostet hat, ein Paradigma zu ändern, aber diese Fälle sind teilweise für ihre Seltenheit bekannt.
@ThorstenS.: Entschuldigung, zu viele Negationen für mich (zu dieser Stunde).
@PristineKavalostka: Mir ist ein solches Beispiel bekannt, und es wäre mir ziemlich peinlich, wenn ich einige Jahre am Überprüfungsprozess teilgenommen hätte. Die Tatsache, dass es * nicht * gut funktioniert (wegen Faulheit, äußerem Druck und was nicht), bedeutet jedoch nicht, dass es * nicht * sollte, und das habe ich behauptet. (Wohl (zumindest) Konferenzpublikationen sind alles andere als wertlos [in CS] und "jeder weiß das". Aber natürlich will niemand offen anerkennen, geschweige denn darauf reagieren, weil * alle * Beteiligten ein Stakeholder sind.)
Ein Beispiel für einen verspotteten Nobelpreisträger ist Shechtman für seine Entdeckung von Quasikristallen. [1] Er verlor seinen Forschungsjob und wurde angewiesen, ein Lehrbuch über Kristalle zu lesen, ähnlich dem, was Ihnen gesagt wurde. Es braucht Ausdauer, um die Leute 1 zu 1 zu überzeugen und das richtige Journal für die Veröffentlichung zu finden. [1] http://www.bloomberg.com/news/2011-10-05/technion-s-shechtman-wins-chemistry-nobel -für-Entdeckung-von-Quasikristallen.html
@JoelReyesNoche: Sie haben über Columbus gelacht ... und sie hatten Recht und Columbus hatte Unrecht. (Sie alle wussten, dass die Erde rund ist, Columbus nahm nur an, dass sie viermal kleiner ist und so den Ozean überqueren kann. Er hatte nur Glück, dass sich dazwischen etwas Land befand, sonst wäre er verhungert, wie andere vorausgesagt hatten.)
Vielleicht fehlt mir etwas, aber wenn das Problem nur darin besteht, dass er kein Feedback hat, kann er dann nicht einfach jemanden bezahlen, der seine Arbeit überprüft? Wenn er ihn bezahlt, muss er ihm natürlich erklären, was los ist ...
siehe auch [Analoga von P vs NP in der Geschichte der Mathematik] (http://mathoverflow.net/questions/160265/analogues-of-p-vs-np-in-the-history-of-mathematics) MO für eine Idee / Analogien, wie es sich auf frühere sehr schwierige Probleme in der Mathematik bezieht; Es gibt andere Fragen, die hier für Peer Review aufgeworfen werden. siehe auch Vorschlag für area51 [stem-review] (http://area51.stackexchange.com/proposals/61370/stemreview); & auch dieser Aufsatz [Math Monster] (http://vzn1.wordpress.com/2012/10/18/math-monster/) enthält viele Hinweise zum P vs NP-Problem, einschließlich über Stapelaustausch-Sites / Fragen hinweg und gibt einige Hinweise wie / warum ist es so schwer.
@Raphael - Das nächste Mal möchten Sie möglicherweise Ihren Kommentar als Antwort anstelle eines Kommentars zur Frage veröffentlichen.
@eykanal: Noted. Ich habe den Kommentar wieder auf [cs.SE] gepostet und nicht angenommen, dass er allgemein gültig ist (hier keine Einschränkung auf CS oder P / NP). Inzwischen gibt es bessere Antworten, also lasse ich es dabei (?).
Wenn Ihr Beweis tatsächlich ist, dass "P = NP" ist, dann ist Ihr Weg vorwärts einfach - schreiben Sie ein Programm, das NP-Probleme tatsächlich in Polynomzeit löst. Sie haben gezeigt, dass Sie [die Verschlüsselung mit öffentlichem Schlüssel aufheben] (http://security.stackexchange.com/a/12807) oder unbekannte [Ramsey-Zahlen] berechnen können (http://en.wikipedia.org/w/). index.php? title = Ramsey% 27s_theorem & action = edit§ion = 5), die Leute werden dir glauben.
@mbeckish Weder würde dies notwendigerweise funktionieren (selbst wenn der Algorithmus eine polynomielle asymtotische Laufzeit hat, ist dies für allgemeine Eingaben möglicherweise überhaupt nicht praktikabel), noch würde dies notwendigerweise jemanden überzeugen (ein Algorithmus kann in einigen Fällen schnell sein, was nicht bedeutet, dass er ausgeführt wird Polynomzeit).
@EricLippert Nicht unbedingt wahr; [Wenn sie zwischen Nominierung und Auszeichnung gestorben sind, können sie auch nach dem Tod noch vergeben werden] (http://en.wikipedia.org/wiki/Nobel_Prize#Posthumous_nominations)
@mbeckish: Raphael ist richtig; Ihre Anforderung ist wahrscheinlich ausreichend, aber nicht erforderlich. Angenommen, P = NP, aber der schnellstmögliche SAT-Löser läuft in einer Zeit von n bis Kajillion. Es könnte immer noch unmöglich sein, alle großen SAT-Probleme in Polynomzeit zu lösen.
@Raphael - Ich bin sicher, wenn es bei genügend Eingaben gut genug funktioniert, würde es die Leute ermutigen, seinen Beweis genauer zu prüfen.
@EricLippert - Richtig. Wirf nur einen ausreichenden Weg nach vorne - definitiv nicht den einzigen und möglicherweise nicht einmal einen machbaren Weg. Wir haben absolut keine Details aus dem OP.
Stellen Sie Ihr Papier einfach auf den entsprechenden Stapeltausch. Es wird früh genug abgeschossen.
@Rinze, [Shechtman] (http://en.wikipedia.org/wiki/Dan_Shechtman) hatte diese Lehrbücher bereits gelesen, er hatte tatsächlich einen Doktortitel in diesem Thema. Sein Beispiel hat also nichts damit zu tun, wenn Leute denen, die kein Lehrbuch gelesen haben, sagen, dass sie eines lesen sollen.
"Es bricht, was die meisten Menschen glauben oder was sie bereits bewiesen haben" klingt ziemlich beeindruckend: Welches Problem haben Sie gelöst?
Das Lösen von P = NP würde die Welt brechen. Ich würde dieses Problem anders lösen als andere große Probleme. Abgesehen davon können wir davon ausgehen, dass Sie P und NP nicht verstehen, wenn dies für Sie nicht offensichtlich ist.
Ein weiterer Link, um die bereits diskutierten Geschichten zu ergänzen und darüber nachzudenken: http://www.sciencealert.com/a-purported-new-mathematics-proof-is-impenetrable-now-what
@djechlin: Der Nachweis, dass P = NP keine praktischen Auswirkungen hat, da der Nachweis, dass es einen Algorithmus gibt, der ein Problem in der Polynomzeit löst, Ihnen nicht hilft, einen Algorithmus zu finden, der das Problem während der Lebensdauer des Universums löst.
Ich wünschte, ich könnte @E.P. ein Recht auf sein Kopfgeld geben. Schön.
Vielleicht hilft es der Community hier, einige "Hinweise" zu geben, welche Methode Sie verwendet haben und welches offene Problem Sie gelöst haben, entweder "gut, es könnte wahr sein" oder "nein, das ist ein häufiger Fehler" zu sagen. Die Fragen baumeln so In der Luft ist es fast unmöglich, nicht zu glauben, dass Ihre Behauptung falsch ist.
Wir haben viele Beispiele, bei denen Rezensenten die Bedeutung ihrer Arbeit nicht erfassen.CNN, einer der wichtigsten Durchbrüche bei maschinellem Lernen und Aufgaben im Zusammenhang mit Computer Vision, wurde in CVPR abgelehnt.oder das dunkle Wissenspapier für Jeffry Hinton, einen der Gründerväter von Deep Learning, war ebenfalls abgelehnt worden.Sehen Sie, wie einflussreich diese beiden Papiere waren (und sind).Also ja, das ist etwas zu erwarten.
Interessant zu wissen, wie diese bahnbrechende Forschung zustande kam?
Ich denke, die Frage ist hier "sollten Sie [allgemein] akzeptieren, was die anderen Ihnen sagen, dass es falsch ist - gegeben, dass sie keine tatsächliche Begründung und einfach bloße Behauptung - über den Glauben - geliefert haben, und auf den Versuch verzichten, zu verstehen, was die Einwände tatsächlich sindund warum ist es falsch? "Oder alternativ: "Wie können Sie etwas Konkreteres darüber gewinnen, was mit Ihrem Papier nicht stimmt, unter der Annahme, dass sie zumindest einen Teil davon lesen und dass ihre Ablehnung darauf beruht, etwas Fragwürdiges zu entdecken, aber dass sie nicht wirklich kommuniziert haben, was."das ist zurück zu dir? "
Machen Sie einen Schritt zurück und fragen Sie sich, ob Sie in einem La-la-Land leben.
Vierzehn antworten:
Pete L. Clark
2014-03-25 07:25:32 UTC
view on stackexchange narkive permalink

Ihre Frage hat einige Probleme. Angesichts einiger der Fragen , die Sie in den letzten Tagen auf anderen SE-Websites gestellt haben, habe ich einige Bedenken, ob Ihre Frage gestellt wird In gutem Glauben, aber für sich genommen ist es eine vernünftige Frage, daher werde ich versuchen, sie zu beantworten.

Das Hauptproblem ist, dass Ihr Schreiben selbst bei dieser relativ einfachen Frage alles andere als klar ist . Wenn Sie in dieser Situation nicht klar schreiben können, sind Ihre Chancen, ein schwieriges Stück Mathematik oder theoretische Informatik zu schreiben, weniger als gut. Zum Beispiel:

Seine Vorgesetzten akzeptieren die Arbeit und sie haben sie in einem bekannten Journal veröffentlicht und werden abgelehnt.

beiseite legen Fragen der Subjekt / Verb-Übereinstimmung und der Konsistenz der Zeitform, der gesamte Satz macht keinen Sinn: Sie können kein Papier veröffentlichen und werden abgelehnt.

Es bricht, was die meisten Leute glauben

Ich weiß nicht, was es bedeutet, "zu brechen, was die meisten Leute glauben".

oder was sie bereits bewiesen haben,

Was? Wollen Sie damit sagen, dass Ihr Beweis anderen nachgewiesenen Ergebnissen widerspricht? Wörtlich genommen würde dies bedeuten, dass Sie gezeigt haben, dass Mathematik inkonsistent ist. In der Praxis kann dies nur bedeuten, dass einige zuvor veröffentlichte Arbeiten falsch sind, wenn Ihr Ergebnis korrekt ist. Wenn dies der Fall ist, müssen Sie sich darüber im Klaren sein und die Mängel in den früheren Arbeiten erklären. Es macht mir Sorgen, dass Sie das nicht wirklich zu glauben scheinen, sondern es einfach als lose Sprache abwerfen.

d. h. er / sie löst das P vs. NP-Problem oder ein anderes bekanntes offenes Problem.

Das Lösen eines offenen Problems würde nicht "das brechen, was die Leute bereits bewiesen haben" ... das bedeutet, dass das Problem offen ist. Auch "P vs. NP-Problem oder ein anderes bekanntes offenes Problem" zu sagen, ist ein seltsames Stück Zurückhaltung: Es gibt kein anderes Problem in der theoretischen Informatik (und nur sehr wenige bis gar kein in der Mathematik insgesamt), das "wie" P ist NP. Es macht also keinen Sinn, dies als Beispiel zu nennen. Es ist wie zu sagen "d. H. Er hat den Heiligen Gral oder eine andere berühmte Tasse gefunden".

In anderen Fragen haben Sie speziell über einen Beweis von P gegen NP gesprochen und sich dann nach der Befragung davon zurückgezogen. Diese Art von Schwanken über das, was Sie getan haben, ist eine rote Fahne der "Verschrobenheit", die Profis vorsichtig machen wird.

Die Prüfer lehnen seine Arbeit ohne Begründung nachdrücklich ab und sagten, dass das Ergebnis falsch sein muss.

Zu sagen, dass das Ergebnis falsch sein muss, ist Dies ist nicht nur eine Rechtfertigung für die Ablehnung, sondern auch die beste Rechtfertigung. Kein professioneller Rezensent wird leichtfertig sagen, dass etwas nicht stimmt. Fast jeder Rezensent, der dies sagt, weist auf mindestens einen bestimmten Fehler hin. Wenn dies nicht der Fall ist, bedeutet dies in der Praxis mit ziemlicher Sicherheit, dass das gesamte Dokument für sie nicht sinnvoll genug war, um genauer zu sein.

Wenn Ihr Berater die Arbeit annimmt, lehnen die Prüfer die Arbeit ab, ohne die Fehler zu erklären (es ist das "beste" Journal in seiner Domäne). Was muss er dann tun? / p>

Wenn Sie ein Papier an das Top-Journal in Ihrem Bereich senden, in dem Sie eine Lösung für das Top-Problem in Ihrem Bereich beanspruchen, und Ihr Papier keinen Sinn ergibt oder nicht einmal ein korrektes Verständnis des Problems aufweist, sind die Redakteure wahrscheinlich nicht viel Zeit als Antwort verbringen wollen. Wenn Sie jedoch aufrichtig daran interessiert sind, ihr Fachwissen zu erhalten, erscheint es sinnvoll, sehr höflich zurückzuschreiben und nach weiteren Einzelheiten zu dem Fehler zu fragen. Wenn Ihre Antwort in irgendeiner Weise argumentativ ist, riskieren Sie, dass die Redaktion denkt, Sie würden sie weiterhin ad infinitum verfolgen, und irgendwann müssen sie aufhören zu antworten. Sie sollten also zurückschreiben und sagen, dass Sie nicht in Betracht ziehen, das Papier erneut in diesem Tagebuch einzureichen, aber für Ihren eigenen Fortschritt wäre es äußerst hilfreich zu wissen, was daran falsch ist. Sie können auch erwähnen, dass Ihr Vorgesetzter das Papier für korrekt befunden hat.

Tatsächlich könnten Sie von Ihrem Vorgesetzten weitere Hilfe dazu erhalten. Wenn Sie das P vs. NP-Problem oder ein anderes bekanntes offenes Problem wirklich gelöst haben und Ihr Vorgesetzter der Ansicht ist, dass Ihre Lösung korrekt ist, warum bewegt Ihr Vorgesetzter dann nicht Himmel und Erde, um sicherzugehen, dass Ihre Arbeit die Aufmerksamkeit erhält, die sie verdient ? Das passt nicht zusammen. Die zwei möglichen Erklärungen scheinen zu sein: (i) Ihr Vorgesetzter ist zu höflich mit Ihnen: Er glaubt nicht wirklich, dass Sie P gegen NP gelöst haben; und (ii) die Imprimatur Ihres Beraters hat in der Community keinerlei Gewicht. Letzteres bedeutet leider, dass seine / ihre Meinung zur Richtigkeit Ihrer Arbeit nicht viel wert ist.

Ein guter Weg, um herauszufinden, ob es (i), (ii) oder - ich gebe zu, dass alles möglich ist! Vielleicht ignoriert das Top-Journal in Ihrem Bereich Ihre revolutionäre Arbeit zu Unrecht - es ist die Hilfe Ihres Beraters, um ein anderes Fakultätsmitglied zur Bewertung der Arbeit zu bewegen, vorzugsweise jemanden in der Abteilung, mit dem Sie kürzlich sprechen können.

Schließlich scheinen Sie echte Bedenken zu haben, dass eine unbekannte Person, die ein berühmtes Problem löst, irgendwie nicht zählt. Dies ist wirklich nicht die Art und Weise, wie die Wissenschaft arbeitet, vorausgesetzt, die unbekannte Person ist in der Lage, die Arbeit auf eine Weise zu präsentieren, die für die Experten sinnvoll ist (und wenn nicht, was für eine Schande, aber was könnte man sonst noch erwarten?). Haben Sie von dem jüngsten Beispiel von Yitang Zhang gehört? Zhang war Dozent an der Universität von New Hampshire, als er die mathematische Welt verblüffte, indem er die Existenz begrenzter Hauptlücken nachwies. Er reichte seine Arbeit bei der besten mathematischen Zeitschrift ein ... und nach allen Berichten akzeptierten sie sie mit ungewöhnlicher Geschwindigkeit . Mit anderen Worten, sie erhielten ein Papier von jemandem, von dem sie wahrscheinlich noch nie gehört hatten, schauten es sich schnell an und stellten fest, dass es ein plausibler Angriff auf ein großes offenes Problem war, und als Ergebnis sprangen sie viel schneller und gründlicher in Aktion als für die meisten Einsendungen bekommen sie. Dies ist eine erstaunliche, aber wahre Geschichte, die zeigt, wie die Community auf eine echte Situation wie diese reagiert.

Da ich selbst kein englischer Muttersprachler bin, neige ich dazu, das Schreiben eines nicht englischen Muttersprachlers toleranter zu gestalten. Y. Zhang ist in diesem Fall möglicherweise kein gutes Beispiel. Er ging 1985 in die USA und promovierte in Mathematik in den USA. Er lebt seitdem in den USA. Soweit ich weiß, hat Zhang kein Problem mit Englisch, während das OP möglicherweise ernsthafte Probleme mit der englischen Sprache hat. Ich stimme jedoch vielen Teilen Ihrer Antwort zu.
@scaaahu: Nicht gut Englisch sprechen zu können, ist kein Charakterfehler. Es ist jedoch ein Problem, wenn Englisch die Sprache ist, in der Sie Ihre Arbeiten schreiben. Es ist auch zu unterscheiden, ob Sie eine Sprache nicht perfekt sprechen oder sich schlecht ausdrücken. In meiner Antwort habe ich versucht, in Fragen der Grammatik und des Gebrauchs nicht viel zu harfen.
Nun, in _His / Her Supervisor (s) akzeptieren die Arbeit und sie ** veröffentlicht ** es in einem sehr bekannten Journal_, meine Vermutung ist, dass das Wort _published_ der Missbrauch des eingereichten Wortes sein könnte.
@scaaahu: Sicher, das OP hat hier völlig das falsche Wort verwendet. Auch dies ist kein Verbrechen, aber dies in der akademischen Arbeit zu tun, könnte sicherlich zu Unverständnis und damit zu Ablehnung führen. Betrachten Sie es so: Wäre es nicht schön, wenn Sprachprobleme die Hauptursache für die Schwierigkeiten des OP wären? Wenn ja, können sie überwunden werden * vorausgesetzt, sie werden anerkannt und angesprochen *. Jemandem nicht zu sagen, wenn sein Schreiben unklar ist, tut ihm meiner Meinung nach keinen Gefallen.
@scaaahu Wenn Sie glauben, mit Ihrem Papier Millionen von Dollar zu verdienen, geben Sie doch ein paar Hundert für einen professionellen Redakteur zum Polieren der Sprache aus.
@Raphael Ich kenne den Standort des OP nicht, daher kann ich diese Frage für ihn nicht beantworten. Ich würde professionelle Editoren finden, die mir helfen, wenn ich er wäre. Ich denke, Sprache ist nur eines seiner Probleme. Es kann auch andere Probleme geben.
@Raphael Ich habe mir die Fragen des OP auf anderen SE-Sites angesehen. Die auf Signal Processing SE sieht in Ordnung aus (ich bin kein Experte. Ich habe nur die Engländer analysiert). Die Fragen zu TCS SE sehen ziemlich schlecht aus. Die auf Math SE ist zu kurz, aber sie hat den Schlüssel zum Problem getroffen. Ich bin verwirrt. Ich denke, er kann bis zu einem gewissen Grad Englisch schreiben. Mein Kommentar oben sollte sagen, dass Englisch wahrscheinlich eines der Probleme ist, die er hat.
"Er hat den Heiligen Gral oder eine andere berühmte Tasse gefunden" - ein fantastischer Satz.
@PeteL.Clark Während sich alle seine Fragen auf P gegen NP beziehen, denke ich, dass er tatsächlich ein Ergebnis für den Shannon-Grenzwert oder den Interferenzkanal haben könnte. Siehe seine Fragen in der Signalverarbeitungsgruppe http://dsp.stackexchange.com/questions/15206/can-we-break-the-shannon-capacity
Herzlichen Glückwunsch, Sie haben (ungewollt, da bin ich mir sicher) das OP davon überzeugt, dass alles in Ordnung sein wird, wenn er sein Englisch korrigiert :) (siehe seinen Kommentar zu meiner Antwort)
@Selfishness: "Wenn eine Person nicht bekannt ist und sehr gute Arbeit geleistet hat, wird sie ihre Arbeit (zumindest zuerst) ignorieren." Angesichts der Tatsache, dass meine Antwort ein spezifisches, aktuelles und klares Gegenbeispiel dazu enthielt, bin ich mir nicht sicher, was ich als Antwort weiter sagen soll.
@ff524: Ja, das war nicht meine Absicht, aber ich denke, es ist in Ordnung: Ob Sprachprobleme die Gesamtheit oder die Hauptlast des Problems sind oder nicht, es wird für das OP immer noch vorteilhaft sein, sie anzugehen. (Tatsächlich kann er nur dann herausfinden, ob dies tatsächlich der Fall ist.)
Ich vermute, dass diese Antwort teilweise deshalb so hoch bewertet wird, weil der Befragte so hochqualifiziert ist, sie zu schreiben, und in der Wissenschaft "auf die harte Tour" etabliert ist. Hier ist ein paar Bkg & eine große Sammlung von Refs zum [Zhang Twin Prime Breakthru] (http://vzn1.wordpress.com/2013/10/04/zhang-twin-prime-breakthru-vs-academic-trackgrind/) & Refs darüber, wie es ist, in der Wissenschaft zu arbeiten & die Herausforderungen dort & Interview mit Zhang etc.
Zumindest habe ich es inhaltlich hoch bewertet - den Autor nicht erkannt und mir nicht die Mühe gemacht, den Zeiger jetzt zu verfolgen.
Alle in Ihrem zweiten Satz verknüpften Fragen wurden für die Aufzeichnung entfernt.
Es könnte erwähnenswert sein, dass die meisten Zeitschriften, die ich bisher eingereicht habe, nur anonymisierte Artikel akzeptieren. Daher hat das Argument, ob Personen, die der Gemeinschaft unbekannt sind, ungerecht behandelt werden, kein Gewicht für oder gegen die Ablehnung.
@EricTobias dies variiert stark je nach Feld. In der theoretischen Informatik sind Single-Blind-Reviews die Norm.
Ich war mir nicht sicher, ob ich +1 für das Lachen über "Holy Grail oder eine andere berühmte Tasse" oder -1 für den Zorn, dem ich sicher begegnen werde, nachdem ich den ganzen Boden gerade mit einem lauten Gelächter geweckt habe. Am Ende Ich entschied mich für -1 + 1 = 0, aber +1 für den Patientenrat.
Wenn die Muttersprache der Person nicht Englisch ist, warum kann sie dann nicht zuerst in ihrer Muttersprache in einer Zeitschrift veröffentlichen? Es gibt nicht englische und nicht US-amerikanische akademische Veröffentlichungen
@DVK: Wenn Ihre Englischkenntnisse nicht stark genug sind und Sie keine Erfahrung im Schreiben auf Englisch haben, ist es wahrscheinlich eine gute Idee, Ihre Arbeit zuerst in Ihrer Muttersprache zu schreiben. Wenn es um Veröffentlichungen geht, ist die Anzahl der Sprachen, in denen seriöse Mathematik- / TCS-Zeitschriften Beiträge akzeptieren, mittlerweile recht gering: Sie können einen Artikel praktisch überall in Englisch oder Französisch einreichen. Einige renommierte internationale Zeitschriften akzeptieren deutsche Papiere [obwohl die meisten deutschen Muttersprachler jetzt alle ihre Papiere auf Englisch schreiben].
Es gibt Länder mit enorm großen und starken mathematischen Gemeinschaften - China, Japan, Russland - in denen praktisch alle seriösen Forscher ihre Arbeiten auf Englisch oder Französisch veröffentlichen. Es gibt * einige * Zeitschriften, die Artikel in anderen Sprachen veröffentlichen - aber diese Artikel werden dann normalerweise nicht von der größeren Community gelesen. Wenn Sie nur etwas veröffentlichen möchten, ok; Wenn Sie jedoch eine Anerkennung für die Lösung eines bekannten Problems wünschen, ist diese Art der Veröffentlichung problematisch: siehe z. http://www.newscientist.com/article/dn24915-kazakh-mathematician-may-have-solved-1-million-puzzle.html#.U-423aPbH-4.
Außerdem erwähnen Sie in allen Hauptstädten, dass es wissenschaftliche Veröffentlichungen außerhalb der USA gibt: Mit wem sprechen Sie? Mehr als die Hälfte aller meiner Veröffentlichungen befindet sich in Zeitschriften, die entweder in anderen Ländern als den USA oder nicht einmal in einem bestimmten Land ansässig sind.
@PeteL.Clark - Es gibt Länder außerhalb der USA. Russland hat mathematische Veröffentlichungen (zumindest hatte ich sie früher zurück, als ich mich darum kümmerte, sie zu überprüfen). Ich bin sicher, China auch. Ich habe nicht gesagt, dass es eine internationale Veröffentlichung sein muss.
@DVK: Wenn Sie nicht "überprüfen" möchten, wie die aktuelle Situation bei mathematischen Veröffentlichungen in anderen Sprachen als Englisch ist, warum kommentieren Sie dies? Ich erklärte, warum es für die Lösung eines berühmten Problems wichtig ist, in einer Sprache zu veröffentlichen, die die Experten auf diesem Gebiet lesen können. Sie überzeugen die Leute nicht, dass Sie kein Trottel sind, indem Sie ein Papier veröffentlichen, das keiner der Experten lesen wird. Wenn Sie nur sagen: "Es gibt Länder außerhalb der USA": Ich verspreche Ihnen, das weiß ich.
@PeteL.Clark - Sie implizieren, dass seine einzige Option darin besteht, in perfektem Englisch zu veröffentlichen. Dies ist NICHT der Fall, es sei denn, die Muttersprache stammt von einem seltenen Stamm in Amazon. Er kann in seinem Heimatland veröffentlichen, und wenn es sich wirklich um ein gültiges Papier handelt, lassen Sie es professionell übersetzen. Es gibt Experten auf diesem Gebiet, die in anderen Sprachen als Englisch lesen, das kann ich Ihnen versichern.
@DVK: Ich impliziere nicht, was Sie vorschlagen. Darüber hinaus habe ich meinen aufrichtigen Rat gegeben, wie man am besten ernst genommen wird, um ein schwieriges Problem zu lösen. Ich habe Ihnen einen speziellen Link zu jemandem gegeben, dessen Arbeit aus genau diesem Grund nicht ernst genommen wird. Da Ihre jüngsten Kommentare nicht mit den Nuancen meiner Antworten übereinstimmen, werde ich in diesem Punkt nicht mehr auf Sie antworten. Wenn Ihr Rat jedoch anders ist, können Sie gerne eine Antwort hinterlassen. Der beste Rat wird durch externe Beweise und persönliche Erfahrungen gestützt, daher wäre ich sehr interessiert, davon zu hören.
Die Links zu den Fragen sind jetzt tot.
In Bezug auf Englisch und Textklarheit kam mir die Frage von OP als hypothetische Frage "Was wäre, wenn jemand ..." anstelle von "Ich behaupte, ich habe ..." vor. Diese Antwort scheint davon auszugehen, dass es sich bei OP tatsächlich um das handeltThema, das er vertrat, anstatt zu fragen, wie die Dinge in dieser Situation laufen (für andere Menschen).
@Mefitico, Ich weiß nicht, ob Ihr Ruf ausreicht, um die Originalversion des Beitrags zu sehen, aber in der ersten Version heißt es im OP "P.S. Dies ist eine $ \ textbf {real} $ Situation".
"Kein professioneller Rezensent wird sagen, dass etwas leichtfertig nicht stimmt."Nach meiner Erfahrung nicht wahr.Ich habe mehrfach behauptet, ein Beweis sei falsch, wenn dies eindeutig nicht der Fall ist (und habe ihn anschließend an anderer Stelle veröffentlicht).Auch sehr kurze Beweise.Es scheint auch in hochrangigen Zeitschriften männlich zu sein.Vielleicht, weil sie am meisten mit schlechten Einsendungen überschwemmt sind, ist es wahrscheinlich eine gute Vermutung, etwas zu erraten, das aufgrund Ihres Darms falsch ist, während faules und inakzeptables Verhalten wahrscheinlich eine gute Vermutung ist.
"In der Praxis kann dies nur bedeuten, dass einige zuvor veröffentlichte Arbeiten falsch sind, wenn Ihr Ergebnis korrekt ist."Glauben Sie fest an die Konsistenz (einiger ausreichend ausdrucksstarker Grundsysteme) der reinen Mathematik?Ansonsten ist mir nicht klar, warum es nur bedeuten könnte, was Sie sagen, dass es bedeuten könnte.
ff524
2014-03-25 06:19:35 UTC
view on stackexchange narkive permalink

Unabhängig davon, ob die Arbeit korrekt ist oder nicht, gilt die folgende Aussage:

Die Beweislast liegt beim Autor, um den Leser vom Ergebnis zu überzeugen.

Die Community (z. B. Redakteure, Rezensenten) ist nicht dafür verantwortlich, Ihre Arbeit zu Ihrer Zufriedenheit zu bewerten. Wenn sich die Prüfer nach Treu und Glauben bemüht haben, Ihr Papier zu lesen, und nicht überzeugt waren, müssen Sie Ihre Argumentation überzeugender machen.

(Dies bedeutet nicht, dass Sie einige geringfügige Änderungen vornehmen und erneut einreichen müssen. Beweisen Sie Ihre Ergebnisse so gründlich und in solch qualvollen Details und mit einem nachweislich hervorragenden Verständnis des Problemkontexts, dass sie unbestreitbar werden. Dann finden Sie einen Weg, um die Ergebnisse auf überzeugende Weise auszudrücken.)

Wenn Dabei finden Sie einen Fehler. Nun, Sie wären in guter Gesellschaft.

+1. "Außerordentliche Ansprüche erfordern einen außerordentlichen Nachweis." Wenn es nicht überzeugend ist, machen Sie es so - und denken Sie daran, dass nur ein Gegenbeispiel erforderlich ist, um zu zeigen, dass Sie das Problem nicht gelöst haben. Sie müssen also wirklich jeden möglichen Randfall berücksichtigen, bevor Sie diesen Anspruch geltend machen können. Wenn es eine Ausnahme gibt, haben Sie sie nicht gelöst, aber möglicherweise haben Sie eine Teilmenge gelöst ... die möglicherweise neue Informationen enthält oder nicht.
@user3439590 Wenn die Rezensenten den Beitrag verstehen, aber der Meinung sind, dass das Englisch verbessert werden sollte, schreiben sie etwas wie "Dieses Papier liefert einen nützlichen Beitrag, hat aber Probleme beim Schreiben in Englisch." Wenn die Rezensenten das nicht geschrieben haben, haben Sie sie nicht davon überzeugt, dass Sie einen nützlichen Beitrag geleistet haben.
@Selfishness_has_equilibrium bedenken Sie jedoch, dass es auch möglich ist, dass die englische Barriere dazu führt, dass Sie einige subtile Dinge mit einigen Definitionen und Begriffen nicht verstehen. Was weiter leicht zur Lösung einer etwas anderen Version des Problems führen könnte ... Und insbesondere in der Graphentheorie ist manchmal ein kleines Detail der Unterschied zwischen einem offenen Problem und einer einfachen Übung in einem Einführungskurs ....
@Selfishness_has_equilibrium, Wenn schlechte Englischkenntnisse die Prüfer daran hindern, zu verstehen, was Sie sagen möchten, werden sie dies natürlich ablehnen. Das gilt doppelt für außergewöhnliche Ansprüche (und dreifach, wenn Sie Verfolgung beanspruchen oder sich mit Einstein vergleichen). Ich würde vorschlagen, dass Sie mit jemandem zusammenarbeiten, der über ausgezeichnete Englischkenntnisse verfügt, um Ihre Arbeit zu überprüfen / auf ihre Gesundheit zu überprüfen und sie auf Lesbarkeit zu bearbeiten. Die Kosten können sein, dass Sie sie (abhängig davon, wie viel sie beitragen) als Co-Autor auflisten müssen.
"Wenn Sie dabei einen Fehler finden, sind Sie in guter Gesellschaft." Erinnerte mich an [diesen Nachrichtenbericht] (http://www.youtube.com/watch?v=nQGvXx4rfUY) (Youtube)
Das Problem ist, weil Sie nicht wissen, was falsch ist oder warum Ihr Beweis nicht überzeugend ist, dann werden Sie effektiv zufällig erraten, und zufälliges Raten scheint mir in Widerspruch zu anderen Prinzipien zu geraten, wie "nicht alle zu verschwenden"Zeit".Obwohl ich nehme an, wenn sie es Ihnen nicht sagen wollen, dann raten Sie nach dem Zufallsprinzip und "verschwenden" Sie ihre Zeit mit 3-4 gründlich überarbeiteten Einsendungen, bevor Sie aufgeben, denn Sie müssen den Widerspruch irgendwie lösen und das bedeutet Siemuss ein Prinzip xoder das andere verbrennen und es scheint besser, das kleinere als das größere zu verbrennen.
@keshlam: "Außerordentliche Ansprüche erfordern einen außerordentlichen Nachweis." - vereinbart. "Wenn es nicht überzeugt, machen Sie es so" - Einverstanden.Das Problem ist, Sie müssen zuerst wissen, warum es nicht überzeugend ist, und es ist unmöglich, die Gedanken von jemandem zu lesen, der Ihnen buchstäblich nichts anderes gibt, mit dem Sie arbeiten können, als "Es ist falsch".Ich glaube, die Frage von OP bezieht sich ausschließlich auf diese Komponente des Mangels an Wissen.Wie können Sie Ihre Arbeit verbessern - oder sogar Ihre eigene Torheit vernünftigerweise ablehnen -, wenn Sie überhaupt nichts mehr zu tun haben, um Ihnen zu sagen, was daran falsch ist?
Sie bekommen jemanden, der Sie unterrichtet.Das beginnt damit, dass Sie zugeben, dass Sie möglicherweise völlig falsch liegen.Wenn Sie es sich nicht leisten können, Unterricht zu nehmen, zu prüfen und / oder Empfehlungen für aktuelle Lehrbücher zu erhalten, und Nachhilfe zu beauftragen, wenn Sie nicht weiterkommen.Wenn sich diese Mühe nicht lohnt, warum sollte jemand zuhören?
Kaveh
2014-03-27 03:17:08 UTC
view on stackexchange narkive permalink

Stellen Sie zunächst sicher, dass Sie nicht wirklich eine Kurbel sind, bevor Sie versuchen, andere zu überzeugen. Lesen Sie diese allgemeinen Merkmale von Kurbeln. Wenn sie sich an Sie wenden, holen Sie sich professionelle Hilfe.

Für den Rest der Antwort gehe ich davon aus, dass Sie ein berühmtes offenes Problem wirklich gelöst haben. Im Folgenden bezieht sich "er" auf eine typische Behauptung eines Nicht-Experten um eine Lösung für ein berühmtes offenes Problem zu haben und "sie" bezieht sich auf eine Expertin in diesem Thema.

  1. Es gibt keine einfache Verknüpfung für Sie! Wenn Sie nach einer einfachen Abkürzung suchen, um Ihre Lösung von einem Experten überprüfen zu lassen, ist diese Antwort nicht für Sie und ich kann Ihnen versichern, dass das, was Sie wollen, nicht passieren wird.

  2. Verstehen Sie die Größe Ihres Anspruchs!
    ZB Wenn Sie behaupten, dass ein Beweis für P nicht gleich NP ist, dann sind Sie der Typ, der behauptet, ein Design für eine Rakete zu haben, die mit der aktuellen Technologie und den Ressourcen gebaut werden kann, um einen Menschen sicher nach Andromeda und zurück zu bringen Experten haben es schwer, einen Menschen zum Mars zu schicken. Wenn Sie behaupten, dass ein Beweis für P gleich NP ist, dann sind Sie der Typ, der behauptet, eine Zeitreisemaschine zu haben.

  3. Verstehen Sie, warum Experten sind Nicht bereit, Nicht-Experten direkt einzubeziehen.
    Viele Experten wären interessiert, über wichtige Fortschritte auf ihrem Gebiet informiert zu werden. Z.B. Es gibt Komplexitätstheoretiker, die jedes P vs. NP-bezogene Papier lesen, das auf arXiv veröffentlicht wurde (arXiv hat eine sehr milde Akzeptanzpolitik in Bezug auf P vs. NP-Ansprüche). Sie werden es definitiv tun Lassen Sie andere Experten wissen, wenn sie etwas Interessantes bemerken. Aber

  4. Sie sind nicht der einzige mit solchen Behauptungen.
    Es gibt Tausende von Menschen, die regelmäßig solche Behauptungen aufstellen.

  5. Alle vorherigen hatten unter trivialen Problemen zu leiden, die kein Experte gemacht hätte.
    Es ist Ihre Aufgabe, zu zeigen, dass Sie keiner von ihnen sind.

  6. Ihre Zeit ist wertvoll.
    Für die meisten ist es nicht wirklich monetär. Aber ich denke, einige Zahlen zu geben wäre hilfreich. An meiner Universität erhält ein Doktorand mehr als 40 US-Dollar pro Stunde, um einfache Aufgaben für Studenten zu kennzeichnen. Dies ist nichts im Vergleich zu dem, was ein Experte für die Beratung in der Branche verlangen könnte.

  7. Nichtfachleuten fehlen häufig grundlegende Fähigkeiten und Kenntnisse, um ihre Antworten zu verstehen.
    ZB Ihm fehlt die mathematische Reife, er kennt keine grundlegenden Definitionen und Begriffe usw. Es ist nicht ungewöhnlich, dass ein Experte einem Nicht-Experten sagt, was er hat, ist kein Beweis. Sie meint nicht, dass der Beweis falsch ist, sie meint, dass es nicht einmal ein Beweis in dem Sinne ist, dass ein Apfel kein Beweis ist. Er versteht nicht, wenn ihm gesagt wird, dass es " nicht einmal falsch!" Ist ". Um ihm ihre Antwort verständlich zu machen, müsste sie ihm die erforderlichen Fähigkeiten und Kenntnisse beibringen, zu viel Arbeit, um ihn davon zu überzeugen, dass er keine Lösung hat. Oft ist er weder geduldig noch interessiert am Lernen (z. B. Lesen eines Lehrbuchs) nur an einer Bestätigung dessen interessiert, was er für eine Lösung hält. In diesem Fall viel zu viel Arbeit.

  8. Es ist oft unmöglich, ihn zufrieden zu stellen.
    Aufgrund der oben genannten Punkte besteht er oft auf der Gültigkeit seiner Behauptung, selbst nachdem sie ihm gesagt hat, dass dies nicht der Fall ist. Zu anderen Zeiten, in denen er die Antwort versteht, betrachtet er sie als einen einfachen, leicht zu behebenden Fehler, nicht als einen grundlegenden. Er versucht, sie zu beheben und sie dazu zu bringen, sie zu überprüfen. Dies führt zum Hin und Her.

  9. Er unterschätzt die erforderliche Zeit und Mühe von ihrer Seite, um seine Forderung zu beantworten.
    Er hält es für eine einfache Aufgabe für sie, seine Forderung zu beantworten. Z.B. Er erwartet von ihr, dass sie ihm ein Gegenbeispiel gibt, bei dem sein Algorithmus fehlschlägt. Ein Gegenbeispiel für einen Algorithmus zu finden, ist eine sehr schwierige Aufgabe (wie jeder wissen würde, der Algorithmen für Bachelor-Studiengänge oder Aufgaben der Komplexitätstheorie markiert hat). Noch schwieriger ist es, eine Erklärung zu finden, warum eine Idee grundlegend fehlerhaft ist und nicht funktionieren kann.

  • Er versteht nicht, dass es kein Rätsel ist.
    Sie interessiert sich nicht nur um ihrer selbst willen für die Frage Die Lösung der Frage wird mit großen Fortschritten auf ihrem Gebiet einhergehen Komplexitätstheoretiker interessieren sich nicht nur um ihrer selbst willen für P vs. NP. Sie erwarten, dass die Lösung für P vs. NP mit großen Fortschritten in unserem Verständnis der Natur effizienter Berechnungen und ihrer Einschränkungen einhergeht. Oft versteht er dies nicht stellt sich die Frage als ein Spiel oder Puzzle vor, von dem er glaubt, dass er gewonnen hat, und das ist es. Diese Einstellung ist für Experten frustrierend.

  • ol>

    Hier sind einige Tipps :

  1. Sei demütig.
    Es ist viel einfacher, sie dazu zu bringen, sich deine Lösung anzusehen, wenn du wirklich bescheiden und lernbegierig bist und Akzeptieren, wenn Ihnen mitgeteilt wird, dass Sie falsch liegen.

  2. Stellen Sie sicher, dass Sie verstehen, was zur Lösung der Frage erforderlich ist.
    ZB Verstehen Sie, dass ein Programm, das ein NP-vollständiges Problem effizient zu lösen scheint, kein Beweis ist, verstehen Sie, dass eine Idee keinen Beweis liefert, stellen Sie sicher, dass Sie die Definitionen und die Terminologie verstehen usw.

  3. Kennen Sie die Grundlagen.
    Ich wiederhole dies immer wieder: Lesen Sie ein gutes Lehrbuch zum Thema und lösen Sie die Übungen. Es ist vorteilhaft für Sie, da Sie mehr wissen und überzeugender sein werden. Es ist vorteilhaft für sie, weil Sie ihre Zeit nicht mit einfachen Fehlern verschwenden werden, die Sie selbst bemerkt hätten, wenn Sie ein gutes Lehrbuch gelesen hätten. Es ist ärgerlich, mit Leuten umzugehen, die behaupten, P gegen NP gelöst zu haben, aber wiederholt grundlegende Fehler machen, die ein guter Student, der einen Grundkurs zu diesem Thema belegt hat, nicht machen wird.

  4. Verwenden Sie Ihren richtigen Namen.
    Wenn Sie Ihren richtigen Namen nicht verwenden, bedeutet dies, dass Sie versuchen, mögliche negative Folgen einer falschen Behauptung zu vermeiden. Die Verwendung Ihres richtigen Namens zeigt an, dass Sie sicher genug sind, potenzielle negative berufliche Konsequenzen zu tragen, wenn Sie sich irren, sodass Sie ernst genommen werden können. Wenn Sie sich über Ihren Anspruch nicht ganz sicher sind, verschwenden Sie nicht ihre Zeit.

    li

    Scheuen Sie sich nicht vor der Arbeit. Machen Sie Ihren Beitrag, bevor Sie Hilfe von anderen erwarten.
    Wenn Sie möchten, dass sie sich Ihre Lösung ansieht, sollten Sie zehnmal mehr Zeit und Mühe aufwenden, als sie Ihnen helfen wird. Für Ansprüche über P vs. NP müssen Sie viel mehr tun.

  5. Sie erhalten nicht mehr als eine Chance.
    Lassen Sie es zählen . Wenn sie auf der ersten Seite Ihres Papiers einen dummen Fehler oder einen grundlegenden Fehler findet (z. B. kennen Sie nicht einmal die Definitionen von P und NP), ist sie für immer mit Ihren Behauptungen fertig.

  6. Verstehen Sie die bekannten Hindernisse für die Lösung der Frage und warum sie nicht für Ihre Lösung gelten.
    ZB Wenn Sie behaupten, P sei nicht gleich NP, sollten Sie eine gute Vorstellung davon haben, warum Relaltivierungs- und natürliche Beweisbarrieren für Ihre Lösung nicht gelten. In ähnlicher Weise, wenn Sie behaupten, P sei gleich NP.

  7. Versuchen Sie, einfachere, akzeptablere Ansprüche zu beweisen.
    ZB. Wenn Sie einen Beweis dafür haben, dass P gleich NP ist, sollten Sie auch einen Beweis für einfachere, schwächere Hauptergebnisse haben, wie Factoring in P. Wenn Sie einen sauberen Beweis für solche Behauptungen extrahieren können, können Sie zuerst versuchen, sie zu veröffentlichen. Solche Ergebnisse können Es ist viel einfacher, sich verifizieren zu lassen, da dies als wahrscheinlicher angesehen wird.

  8. Stellen Sie sicher, dass Ihre Lösung nicht zu stark ist.
    Mit anderen Worten: Stellen Sie sicher, dass dies nicht anderen bekannten Ergebnissen widerspricht. Z.B. Wenn Ihr Argument für P gleich NP ist, würde dies auch zeigen, dass P gleich ExpTime ist (von dem wir wissen, dass es falsch ist), dann sind Sie in Schwierigkeiten (Scott Aaronson erwähnt in seinem Blog-Beitrag Acht weitere Fälle von zu starken Ergebnissen Unterzeichnet einen behaupteten P ≠ NP-Beweis ist falsch).

  9. Überprüfen Sie Ihre Lösung.
    Stellen Sie sicher, dass keine Fehler vorliegen. Alle Schritte sollten leicht zu erkennen sein, um den vorherigen zu folgen. Stellen Sie sicher, dass Sie zu keinem Zeitpunkt zusätzliche Annahmen treffen.

  10. Überprüfen Sie Ihre Lösung erneut.
    Legen Sie Ihren Beweis zwei Wochen oder länger vollständig beiseite . Denk nicht drüber nach. Gehen Sie dann zurück und überprüfen Sie es mit neuem Verstand, als würden Sie die Lösung eines anderen prüfen.

  11. Erstellen Sie Beweise für Ihre Behauptungen.
    ZB. Wenn Sie einen wirklich effizienten Algorithmus haben (dh seine Laufzeit ist ein Polynom mit kleinen Konstanten), von dem Sie bewiesen haben, dass er ein NP-vollständiges Problem löst, sollte es keine schwierige Aufgabe sein, den Zustand zu überwinden. of-art SAT-Löser oder verschiedene kryptografische Protokolle basierend auf Härte-Vermutungen zu brechen (diese Vermutungen sind falsch, wenn P gleich NP ist).

  12. Schreiben Sie einfach zu lesende, übersichtliche, saubere Zusammenfassungen und Einführungen.
    Geben Sie keine unnötigen Hintergründe / Geschichten / philosophischen Konsequenzen / Diskussionen über Wichtigkeiten / allgemeine Kommentare an. Es ist ein berühmtes offenes Problem, jeder Experte kennt seine Bedeutung. Speichern Sie sie für Ihre endgültige Version. Jetzt sollten Sie sich darauf konzentrieren, sie davon zu überzeugen, dass Ihre Behauptung richtig ist. Level Erklärung Ihrer Lösung. Es sollte auch erklären, warum bekannte Hindernisse für Ihre Lösung nicht zutreffen. Es sollte auch andere Beweise enthalten, die die Richtigkeit Ihrer Behauptung belegen können. Wenn Sie versagen, liest der Leser wahrscheinlich nicht weiter.

  13. Stellen Sie sicher, dass der Rest Ihres Papiers mit Ihrer Zusammenfassung und Einführung übereinstimmt.
    Wenn Wenn Sie versagen, wird der Leser wahrscheinlich nicht weiterlesen.

  14. Stellen Sie sicher, dass alle Details in Ihrem Artikel korrekt sind.
    Befolgen Sie die Standardstruktur von Artikel in diesem Thema. Überprüfen Sie einige berühmte, gut geschriebene Artikel in diesem Bereich, die wichtige offene Probleme gelöst haben. Alle Definitionen sollten klar, leicht verständlich und streng sein. Jeder Satz (Lemma usw.) sollte klar und streng formuliert sein, und der Beweis für jeden von ihnen sollte ihrer Aussage folgen. Sie sollte in der Lage sein, anhand der vorherigen Schritte, Definitionen und Deckspelzen ohne allzu großen Aufwand zu erkennen, warum jede Behauptung im Beweis korrekt ist. Wenn Sie versagen, liest der Leser wahrscheinlich nicht weiter.

  15. Lassen Sie einen allgemeinen Experten, der persönlich weiß, dass Sie Ihre Lösung überprüfen. stark>
    Ich gehe davon aus, dass Sie persönlich keinen Experten auf dem Gebiet der Frage kennen. Je näher der allgemeine Experte dem Bereich der Frage ist, desto besser wird es sein. Z.B. Für P vs. NP können Sie einen Mathematiker, vorzugsweise einen theoretischen Informatiker, fragen. Die Meinung von Personen, die keine Experten auf diesem Gebiet sind, hat möglicherweise nicht viel Gewicht, stellt jedoch sicher, dass Sie keinen einfachen Fehler machen.
    Verstehen Sie dass zu diesem Zeitpunkt jemand, der Sie nicht persönlich kennt, keinen Grund hat, Ihre Lösung zu überprüfen.

  16. Haben Sie einen anderen allgemeinen Experten, der Sie kennt persönlich überprüfen Sie Ihre Lösung.
    Rom wurde nicht an einem Tag erbaut. Sie müssen nach und nach Vertrauen in Ihre Lösung aufbauen. Wer Sie überzeugt, kann zu Ihren Brücken werden, um die Experten zu erreichen.

  17. Wenn sie überzeugt sind, bitten Sie sie, Ihre zu zeigen Lösung für einen Experten, den sie kennen.
    ZB Bitten Sie sie, dies einem ihnen bekannten Komplexitätstheoretiker zu zeigen. An diesem Punkt ist es weniger wahrscheinlich, dass Sie einen grundlegenden Fehler machen, und Sie haben gute Beweise, um Ihre Behauptung zu stützen. Ihre Lösung erfordert jetzt das Fachwissen eines Experten für das Thema.

  18. Wenn sie überzeugt ist, wird sie es definitiv anderen Experten zeigen.
    Nachrichten über wichtige Fortschritte in einem Bereich werden sich unter den Experten in diesem Bereich sehr schnell verbreiten. Andere Experten (Komplexitätstheoretiker im Fall P vs. NP) werden Ihre Lösung unabhängig überprüfen. Wenn sie überzeugt sind, erhalten Sie wahrscheinlich eine Einladung, Ihre Arbeit in einem berühmten Journal einzureichen (so etwas wie JACM im Fall von P vs. NP).

  19. Behaupten Sie nicht, ein berühmtes offenes Problem mehr als einmal zu lösen.
    Wie ich oben geschrieben habe, erhalten Sie nicht mehr als eine Chance! Sie haben kein Recht, sie zu fragen, was mit Ihrer festen Lösung nicht stimmt, wenn Sie einen Fehler gemacht haben. (Die Ausnahme ist, wenn sie Sie explizit auffordert, zu versuchen, Ihre Lösung zu reparieren und die feste zu senden Version zurück zu ihr.)

  20. Erwarten Sie keine Erklärung dafür, warum Ihre Idee nicht funktionieren kann.
    Es ist unwahrscheinlich, dass jemand dazu in der Lage ist formal zu zeigen, dass eine informelle Idee nicht funktionieren kann. Wenn die Idee formal genug ist, kann der Grund, warum sie nicht funktionieren kann, ein neues interessantes Ergebnis für sich sein. Der Nachweis solcher Ergebnisse kann jedoch noch schwieriger sein als die Lösung der ursprünglichen Frage. Wenn Sie im Fall von P vs. NP behaupten, einen effizienten Algorithmus für ein NP-schweres Problem zu haben, sollten Sie nicht erwarten, dass sie eine Eingabe findet, bei der Ihr Algorithmus fehlschlägt.

  21. ol>

    Zusammenfassend:

    Verstehen Sie, dass sie nicht verpflichtet ist, Ihnen zu helfen. Wenn sie Ihnen hilft, tut sie dies aus Großzügigkeit. Sie hat das Recht, es jederzeit zu beenden Sie freut sich ohne jede Erklärung. Denken Sie an ihre Zeit, verschwenden Sie sie nicht für das, was Sie selbst hätten tun können / sollen, versuchen Sie, ihre Arbeit so einfach wie möglich zu machen, und tun Sie nichts, was sie bereuen würde, es versucht zu haben um Ihnen zu helfen.

Was meinst du mit "verstehe, dass es kein Rätsel ist"?
@Jack, hier ist, was ich meinte: Einige Leute behandeln P gegen NP wie ein einstufiges Computerspiel mit einer Ja- oder Nein-Antwort, die sie gewinnen müssen. Wir (Komplexitätstheoretiker) kümmern uns um P vs. NP, weil wir glauben, dass die Klärung der Frage mit erheblichen Fortschritten in unserem Verständnis der Natur effizienter Berechnungen und ihrer Grenzen einhergehen wird. Wir kümmern uns nicht nur um ihrer selbst willen darum. Wie Scott einmal schrieb: "_ [Wir] nehmen gerne P vs. NP als unser" Flaggschiff-Beispiel "für eine große Klasse von Fragen darüber, was für Computer machbar ist und was nicht, von denen wir keine zu beantworten wissen." ""
"Wenn Sie einen Algorithmus haben, von dem Sie bewiesen haben, dass er ein NP-vollständiges Problem löst, sollte es keine schwierige Aufgabe sein, ihn zu überwinden ..." - dies kann ein schlechter Rat sein. Es gibt keinen Grund zu der Annahme, dass ein solcher Algorithmus "wirklich" effizient sein sollte: Er kann bei allen Eingaben, die wir speichern können, eine miserable Leistung aufweisen. Ansonsten gute Antwort. Es zeigt perfekt, dass es außerordentlicher Anstrengungen bedarf, um ein außergewöhnliches Problem zu lösen. Natürlich wäre eine Kurbel a) nicht in der Lage, sich selbst zu diagnostizieren und b) viele Ihrer Punkte zu widerlegen, weil sie diese Verschwörungstheorien haben. (Sie * nehmen * an, dass jeder freundlich ist.)
+1: Was für eine außergewöhnlich detaillierte und hilfreiche Antwort. Ich hoffe, das OP weiß die Großzügigkeit von Zeit und Geist zu schätzen.
@Raphael, Sie haben im Prinzip Recht (obwohl es in der Praxis selten vorkommt), habe ich versucht, die Aussage präziser zu machen.
@Kaveh: Ich denke, es ist denkbar, dass ein Zeuge von P = NP (falls vorhanden) * genau * ein solcher Algorithmus ist. Es würde definitiv erklären, warum bisher niemand einen gefunden hat. Vielen Dank für die Änderung.
Ein weiterer Punkt auf der Checkliste könnte sein, dass Sie über das Problem mit der Notation sprechen sollten, die am meisten Standard ist, eine Erklärung des offenen Problems, in dem diese Notation verwendet wird, sehr standardisiert zitieren und keine Haufen Ihrer eigenen Notation für Dinge einführen sollten bis sie eindeutig notwendig werden.
Es ist nicht wahr, dass ein asymptotisch besserer Algorithmus für ein schwieriges Problem es einfach macht, vorhandene Software-Implementierungen anderer Algorithmen zu schlagen. Zum Beispiel gibt es diesen linearen Zeitalgorithmus für die Polygon-Triangulation, der asymptotisch besser ist als die einfache Methode O (n log (n)) - aber viel Glück beim Implementieren in einer angemessenen Zeitspanne! Ganz zu schweigen von der Tatsache, dass vorhandene Implementierungen häufig viele Optimierungen und nützliche Verknüpfungen für häufige / kleine Fälle usw. beinhalten. Also - ich würde diesen Vorschlag zurückziehen, es sei denn, OP ist ein erfahrener Programmierer.
Dies ist eine großartige Antwort! Jeder, der an dem Problem arbeitet, sollte dies lesen. Es ist wie bei den anderen, bekannteren Listen, wie man erkennt, ob man wie eine Kurbel aussieht, im Internet, nur viel konstruktiver und hilfreicher. Ich hoffe, dass diese Antwort ein großes Publikum in der Zielgruppe findet, obwohl ich nicht wissen würde, wie ich das am besten erreichen kann. Das Versprechen "Ich gehe davon aus, dass Sie ein berühmtes offenes Problem wirklich gelöst haben", dem Sie gerecht werden, wird hoffentlich sicherstellen, dass sich die Leute Ihren Rat tatsächlich zu Herzen nehmen (insbesondere, weil sie dies in den meisten Fällen natürlich noch nicht getan haben, aber noch nicht dazu bereit sind) Gesicht das).
Ihr Punkt (2) ist großartig.
Diese Antwort ist hervorragend und meine erste Anlaufstelle zu diesem Thema. Hab ein Kopfgeld! (die ich vergeben werde, nachdem die Woche der kostenlosen Werbung vorbei ist)
+1 für die Unterpunkte in 3. Ich denke jedoch nicht, dass der erste Ratschlag zum Lesen der allgemeinen Merkmale von Kurbeln zur Selbstdiagnose effektiv ist. Alle Kurbeln, die ich kenne, würden argumentieren, dass sie nicht auf sie zutreffen (z. B. Dunning-Kruger).
Wenn jedoch jemand bereits nach Erhalt des Papiers gesagt hat, dass es falsch ist, deutet dies darauf hin, dass er mindestens einen tatsächlichen und einen konkreten [wahrscheinlich signifikanten] Fehler darin gesehen hat.Es besteht also keine Notwendigkeit, sie um eine "spezialisierte Bewertung" zu bitten oder "sie dafür zu bezahlen, dass sie sich über Stunden hinweg gabeln".Wenn sie es bekommen haben, es trotzdem überprüft haben und gesagt haben, es sei falsch, hat ihnen etwas einen Tipp gegeben, und was OP wissen möchte, denke ich, ist, was Sie tun, wenn das etwas nicht kommuniziert wurde.Die einzige zusätzliche "Arbeit", die der Rezensent benötigt, wäre, mehr zu kommunizieren und das Papier nicht weiter zu analysieren.
Manchmal ist etwas so inkohärent, dass es nicht möglich ist, auf einen falschen Teil hinzuweisen.Es ist das, was wir "es ist nicht einmal falsch" nennen, "es analysiert nicht einmal".Aufgrund früherer schlechter Erfahrungen mit solchen Autoren befürchtet die Expertin manchmal, auf ein bestimmtes Problem in dieser Situation hinzuweisen, da solche Autoren es oft so behandeln, als sei es ein Problem, es zu beheben und erneut zu versuchen, während das, was sie sagt, nicht "das ist" ist_das_ Problem mit Ihrer Lösung "aber" hier ist nur eines der Probleme mit Ihrer Lösung, und es gibt viele weitere davon ".
Bei mathematischen Problemen ist es möglich, einen formalen Beweis zu haben und ihn mit einem Beweisprüfer wie Coq zu validieren. Ein Autor kann Zeit damit verbringen, den Beweis ohne einen Experten zu formalisieren und zu validieren.Die Berufung auf einen Export ist eine Abkürzung, um die Akzeptanz für einen weniger formalen Beweis zu erhalten.Ein Experte muss die Richtigkeit in kurzer Zeit beurteilen.Einige häufige Probleme sind, dass der Autor die Definitionen falsch versteht oder erhebliche Lücken in seinen Argumenten aufweist.In dem späteren, z.Sie kann nur sagen "A folgt nicht aus B", was bedeutet, dass es eine signifikante Lücke im Beweis gibt (anders als "A-> B ist falsch").
Wie @Kimball sagte, leiden fast alle Kurbeln (mit Ausnahme der wirklich geknackten, die nur Unsinn plappern) unter Mahn-Krüger und werden niemals zugeben, dass sie eine Kurbel sind.Erinnert mich an eine inkonsistente Erweiterung T von PA, die darauf besteht, dass sie selbst konsistent ist, egal wie streng Sie durch T's eigene Annahmen zeigen, dass T falsch ist.
Suresh
2014-03-25 05:28:22 UTC
view on stackexchange narkive permalink

Wenn Ihre Interpretation der Ereignisse lautet: "Ich habe eine herzzerreißende Arbeit von erstaunlichem Genie und das einzige Hindernis für die Akzeptanz ist, dass ich nicht bekannt bin und die Eliten meine Arbeit blockieren", dann werden Sie wahrscheinlich keinen guten Rat bekommen was hier oder anderswo zu tun ist.

Das Problem ist, wie Raphael angibt, dass es zwar möglich ist, dass diese Interpretation korrekt ist, es jedoch weitaus wahrscheinlicher ist, dass Ihr Ergebnis tatsächlich NICHT das offene Hauptproblem löst, das Sie haben denke, es tut.

Wenn Sie zugeben, dass diese Möglichkeit besteht, werden viele Schritte angezeigt, die alle in den sehr guten Links aufgeführt sind. Wenden Sie sich an Personen, die Ihre Arbeit kommentieren könnten, und schauen Sie in der Literatur nach, ob Ansätze wie Ihre bereits ausprobiert wurden und fehlgeschlagen sind. Prüfen Sie, ob Ihre Lösung auch verwandte (einfachere) Probleme löst, und so weiter.

Ja. Nur wenige werden dies erwähnen und vielleicht wird es als tabu angesehen, dies zu tun, aber wie in Ihrer Antwort angedeutet, scheint es in einigen Fällen so, als ob diese grandiosen Behauptungen mit psychischen Symptomen / Problemen korrelieren könnten, z. B. [Größenwahn] (http: //en.wikipedia) .org / wiki / Grandiose_delusions), [Narzissmus] (http://en.wikipedia.org/wiki/Narcissism), [narzisstische Persönlichkeitsstörung] (http://en.wikipedia.org/wiki/Narcissistic_personality_disorder) usw.
Dies kann auch ein Merkmal einer bipolaren Störung sein. (Dies bedeutet nicht, dass ein Forscher mit bipolarer Störung keine hervorragenden Ergebnisse erzielen kann. Es soll nur auf eine rote Fahne hingewiesen werden, die auf ein mögliches Problem hinweist.)
Nate Eldredge
2014-03-25 07:19:45 UTC
view on stackexchange narkive permalink

Was sagt Ihre Beraterin zu all dem?

Wenn sie wirklich glaubt, dass Sie dieses Hauptproblem gelöst haben, sollte sie Berge versetzen, um Sie bei der Veröffentlichung und Verbreitung zu unterstützen. (Es hört sich so an, als ob ihr Name auch darauf steht, also hat sie einen noch größeren Anreiz.) Aber Sie haben die eher lauwarme Formulierung verwendet, die sie "akzeptiert". Holen Sie sie besser zuerst vollständig an Bord oder lassen Sie sie ihre Vorbehalte genauer erläutern (was sich tatsächlich als schwerwiegende Mängel herausstellen kann).

Der Vorteil, den Sie gegenüber der durchschnittlichen Kurbel haben, ist, dass als Student, Sie haben bereits Verbindungen zur wissenschaftlichen Gemeinschaft durch Ihren Berater. Nutzen Sie dies. Wenn Sie und Ihr Berater davon überzeugt sind, dass Ihr Manuskript von bester Qualität ist (siehe Antwort von ff254), veröffentlichen Sie es auf arXiv und verteilen Sie es. Ihre Beraterin kennt sicherlich Experten auf diesem Gebiet und sollte einen ausreichenden Ruf haben, um sie dafür zu interessieren.

Ich bin mir in Ihrem Bereich nicht sicher, aber in der Mathematik tendiert die Community derzeit dazu, Lösungen für wichtige offene Probleme zu finden. Sie senden es nicht einfach an Annals , lassen es von den Schiedsrichtern genehmigen und warten dann einige Monate, bis alle ihre Ausgabe von Annals per E-Mail erhalten und erstaunt sind. Stattdessen veranlassen Sie die Community, es zuerst zu studieren. Sie überzeugen einige Experten, dass es plausibel genug ist, ihre Aufmerksamkeit wert zu sein, und sie betrachten es. Entweder finden sie sofort einen kritischen Fehler (der häufigste Fall), oder sie finden einen geringeren Fehler, den Sie oder eine andere Person beheben, und möglicherweise entwickelt sich allmählich ein Konsens darüber, dass er wahrscheinlich richtig ist. Das ist , wenn Sie es an Annals senden.

Eine Sache, die mich in dem, was Sie geschrieben haben, beunruhigt, ist:

Es bricht, was die meisten Menschen glauben oder was sie bereits bewiesen haben ...

Was ist das? Die Unterscheidung ist entscheidend. Wenn es der Intuition der Menschen widerspricht, legt das die Messlatte ein wenig höher, aber Wissenschaftler sind es gewohnt, überrascht zu sein. Wenn es etwas widerspricht, was zuvor bewiesen wurde, legt das die Messlatte sehr hoch. Sie müssen nicht nur zeigen, dass Ihre Arbeit richtig ist, sondern auch, warum die zuvor akzeptierte Arbeit tatsächlich falsch war. (Sie können nicht einfach sagen: "Meins ist richtig, deshalb müssen sie falsch sein.") Sie haben nichts darüber gesagt, dass Sie das getan haben. (Und wenn Sie in der vorherigen Arbeit keinen Fehler finden können, entspricht Ihre Behauptung tatsächlich der von "Mathematik ist inkonsistent". Die Leiste in dieser ist mehr oder weniger auf dem Mond.)

Ich habe auch eine ähnliche Antwort hinterlassen, sogar bis zu der (unabhängig gewählten) Formulierung "Berge bewegen" gegen "Himmel und Erde bewegen". Ich sollte zwar sagen, dass es ein guter Weg ist, die Community dazu zu bringen, Ihre revolutionäre Arbeit zu studieren, aber ich denke nicht, dass dies immer so geschieht, insbesondere bei Menschen, die nur wenige Verbindungen zu qualifizierten Experten haben. Ich habe in meiner Antwort das (erstaunliche) Beispiel von Yitang Zhang angesprochen. Soweit ich weiß, hat er wirklich isoliert gearbeitet und sein Papier den Annalen vorgelegt, anstatt es viel zu kaufen.
@PeteL.Clark: Heh. Großartige Köpfe usw. Vielen Dank für Ihre nette Antwort, das Beispiel von Zhang und insbesondere für die Dekonstruktion der Frage.
@NateEldredge "Der Balken für [Mathematik inkonsistent beweisen] ist mehr oder weniger auf dem Mond" Beachten Sie, dass Mathematik * nicht * konsistent sein kann, also ist der Glaube, dass Mathematik konsistent ist, auch nur Intuition :) Natürlich die Community ist es nicht gewohnt, * so * überrascht zu sein!
Ihre Antwort erinnert mich an eine [wirklich interessante Geschichte] (https://golem.ph.utexas.edu/category/2011/09/the_inconsistency_of_arithmeti.html). Kurzfassung: Ein seriöser Mathematiker glaubt, einen Beweis dafür gefunden zu haben, dass Mathematik (PA) inkonsistent ist, erklärt ihn klar genug, dass Experten ihn verstehen können, ein Top-Mathematiker liest die Gliederung und entdeckt den Fehler, und der Autor zieht den Beweis zurück. So werden auch wirklich außergewöhnliche Behauptungen betrachtet, wenn sie von einer vernünftigen Person klar und vernünftig erklärt werden.
@IstvanChung gut, technisch gesehen, ist es nicht per Definition konsistent?
@Lohoris Siehe [Gödels zweiter Unvollständigkeitssatz] (http://en.wikipedia.org/wiki/Godel%27s_incompleteness_theorem#Second_incompleteness_theorem).
Es könnte möglich sein, Mathematik als konsistent zu beweisen, aber nur, wenn sie inkonsistent ist
@NoahSnyder: Später Kommentar, aber ich habe seit Monaten / Jahren versucht, diesen Fall / Link aufzuspüren. Gott sei Dank, Sie haben es hier aufgenommen!
rumtscho
2014-03-28 01:52:34 UTC
view on stackexchange narkive permalink

Indem Sie Ihren eigenen Beweis noch stärker angreifen als die anderen.

Im Ernst, es gibt einen Grund, warum Menschen in Ihrer Disziplin seit Jahrhunderten keine Antwort mehr finden konnten. Die a priori Wahrscheinlichkeit, dass Sie falsch liegen, ist so hoch, dass selbst wenn Sie einen gut aussehenden Beweis erstellt haben, die a posteriori Wahrscheinlichkeit, dass Sie richtig liegen, viel zu gering ist. Das heißt, wenn Sie genug über Ihre eigene Disziplin wissen, sollten Sie nicht davon überzeugt sein, dass Sie sie gelöst haben. Angesichts eines Problems, das sich lange Zeit der Lösung widersetzt hat, ist die Überzeugung, dass Sie es gelöst haben, nur weil Sie einen Beweis haben, an den Sie glauben, ein sicheres Zeichen für einen Spinner.

In einem solchen Fall besteht das Nicht-Crackpot-Verhalten darin, den Beweis auseinander zu nehmen, abzuschießen und aus allen möglichen Winkeln in Stücke zu reißen. Dies ist, was Ihre Kollegen tun werden, und dies ist, was sie von Ihnen erwarten werden. Deinen Stolz, deine subjektiven Vorurteile zu vergessen und gnadenlos mit deinem eigenen Ergebnis umzugehen.

Sie glauben Ihnen erst, nachdem Sie mehr Möglichkeiten gefunden haben, Ihr Ergebnis zu widerlegen, als sie sich vorstellen können, sie alle ausprobiert haben und in allen versagt haben. Und Ihr Papier muss klar zeigen, dass Sie dies getan haben. Alles andere bringt dir den Crackpot-Titel.

Einfach genial. Ich hätte es selbst nicht besser formuliert ... die Idee kam mir, als ich mich an meinem ersten "Rande des wissenschaftlichen Durchbruchs" befand; Ich gab mir eine kalte Dusche, eine ernsthafte Schelte, eine starke Kritik am Wert der Lösung - und erst dann wurde mir klar, dass meine Lösung zwar * für das vorliegende Problem irrelevant ist und es nicht löst *, aber ein Potenzial darstellt * andere, verwandte Probleme * zu lösen. Um dies zu erreichen, muss man wissen, wie man versagt ...
mirkastath
2014-03-25 11:24:58 UTC
view on stackexchange narkive permalink

Darf ich zu Nate Eldredges umfassender Antwort hinzufügen, dass es sehr wichtig ist, dass Sie diese Ansichten mit Ihren in Einklang bringen, wenn Ihre Arbeit die in Ihrer Community häufig vertretenen Ansichten erschüttert oder zerstört. Damit meine ich: Zeigen Sie genau, wo sich die Community befindet ist "falsch" oder "nicht genau richtig" und warum. Bieten Sie Gegenbeispiele, Vorhersagen, alles was Sie können.

Relativitätstheorie wäre nirgendwo anders, wenn sie sich nicht auf gute alte Newtonsche Mechaniken reduzieren würde, bei denen letztere perfekt funktionieren!

Und manchmal ist es angebracht zu erklären, warum es * schien *, dass das, was getan wurde, nicht getan werden konnte.
usul
2014-03-25 07:23:21 UTC
view on stackexchange narkive permalink

Einige Ratschläge sind, sehr sorgfältig zu überprüfen, ob die Beweise korrekt sind, den Vorgesetzten um Rat zu fragen und dritte Meinungen einzuholen. Vielleicht hat der Betreuer Kollegen im Forschungsbereich, die bereit wären, den Entwurf zu lesen und konkretes Feedback zu geben.

Wenn das eingereichte Journal gut ist, haben die Gutachter jedoch überhaupt kein nützliches Feedback gegeben Es gibt mit ziemlicher Sicherheit ein Problem mit der Zusammenfassung und Einführung.

Die Zusammenfassung und Einführung sollten die neue Idee verdeutlichen, die diesen "Durchbruch" ermöglicht. Vermutlich haben sich viele in der Vergangenheit diesem Problem genähert und sind gescheitert; Es kann weit verbreitete Überzeugungen darüber geben, warum es schwierig ist, "Hindernisse" für versuchte Beweise zu beweisen, oder vielleicht sogar bekannt. In der Zusammenfassung und Einleitung sollte klar und kurz erwähnt werden, warum solche Überzeugungen, Einwände oder Hindernisse nicht zutreffen oder wie sie überwunden wurden.

Kurz gesagt, die Zusammenfassung und Einleitung müssen dem skeptischen Leser Grund geben, dem Papier zu glauben könnte angesichts des Hintergrundwissens des Lesers richtig sein. In diesem Fall würde ich hoffen, dass die Prüfer zumindest erwähnen, warum sie dem Ergebnis nicht glauben.

Thorsten S.
2014-03-27 22:40:09 UTC
view on stackexchange narkive permalink

Während die meisten Antworten viel Vertrauen in das akademische System zu haben scheinen, möchte ich einen anderen Standpunkt anbieten.

Ich denke, es ist tatsächlich viel schwieriger für einen Unbekannten (für ein bestimmtes Gebiet). der wissenschaftlichen Gemeinschaft eine Lösung zu präsentieren, als normalerweise erwartet.

Wissenschaftler machen Mist und manchmal königlich.
Erstes Beispiel: Das berüchtigte Monty Hall-Problem.
Mehr als 65% aller professionellen Antworten auf Marilyn (mit allen Arten von akademischen Abschlüssen, einschließlich Statistiker), lehnten ihre Antworten nachdrücklich ab, manchmal ohne richtige Hohn- und Verspottungen. Dazu gehörten Paul Erdos und Straight Dope Cecil Adams. Selbst die Mehrheit der Experten kann scheitern.

Zweites Beispiel: Das ebenfalls berüchtigte Neutrino Anomalie. Das Interessante hier ist nicht der Fehler selbst, sondern die Reaktion aufArxiv. Jeder, der es gewagt hätte, vor der Ankündigung Superluminal-Theorien anzubieten, wäre sofort als Relativitätskurbel deklariert worden. Nachdem die Ankündigungspapiere überflutet waren, boten sie alle Arten von Superluminal-Theorien an, die erklärten, was wir jetzt als einfach abad Kabel kennen.

Mit welchen Problemen kann ein Unbekannter konfrontiert sein?

  1. Arxiv. Sie benötigen eine Zugehörigkeit zu einer Universität oder einem Forschungsinstitut und / oder eine Bestätigung eines bekannten Autors. Arxiv kann Ihren Zugriff ohne Erklärung widerrufen oder einschränken. Diese Anforderung gilt auch für voll qualifizierte Wissenschaftler, die in Unternehmen arbeiten.

  2. Zeitschriften. Zu viele Menschen versuchen, ihre Ergebnisse in zu wenigen seriösen Zeitschriften zu veröffentlichen. Zeitschriften sind auch ziemlich schlecht skaliert, man muss lange warten, um veröffentlicht zu werden. Weniger bekannte Zeitschriften haben möglicherweise niedrigere Barrieren, aber Sie haben die reale Gefahr, dass der Beitrag verfehlt wird. Und selbst die unteren Zeitschriften lehnen das Papier möglicherweise ab.

  3. Wissenschaftler. Die Situation ist in verschiedenen Ländern unterschiedlich, aber normalerweise sind Wissenschaftler überarbeitet und unterbezahlt. Sie haben weder die Zeit noch die Ressourcen, um Beiträge mit der sehr geringen Chance zu überprüfen, eine zu erhalten wissenschaftlicher Jackpot.

  4. ol>

    Wenn jemand der Meinung ist, dass der Standpunkt nicht gültig ist, versuchen Sie nur zum Spaß, ein normales Papier unter einem Pseudonym und der Privatadresse bereitzustellen.

    Die einzig gangbare Option, die ich sehe, besteht darin, Kontakt zu den Wissenschaftlern auf dem Gebiet aufzunehmen und zu versuchen, mit ihnen über den Beitrag zu arbeiten, der möglicherweise schwieriger ist als seine Gründe. Die von Kaveh bereitgestellte Liste ist zunächst eine gute Ressource.

Ihre Einschätzung des Monty Hall-Problems ist falsch. Das zugrunde liegende Problem war, dass das Problem nicht genau spezifiziert war. Die subtile Mehrdeutigkeit in der Spezifikation führte zu Meinungsverschiedenheiten zwischen Experten.
@EnergyNumbers Ja, ich habe diese Ausrede gehört. Die ursprüngliche Frage bietet jedoch keine Mehrdeutigkeit. Der Gastgeber kennt die Position des Autos (es ist nicht zufällig), es ist eine Spielshow (er kann die Autotür nicht öffnen) und er wählt * eine andere * Tür mit einem * Leerzeichen *. Und wirklich, was fast alle Kritiker sagten, war * nicht *: "Es hat zwei verschiedene Lösungen, die Antwort hängt von den folgenden Umständen usw. ab." Was sie meistens sagten, war: "Die Chance ist immer 1/2, du dummer Trottel !!" Sowohl Massimo Piattelli-Palmarini als auch Gero von Randow, ein Wissenschaftsjournalist, erklärten die Lösung und die Menschen waren sich immer noch nicht einig.
Um nur eine laufende Diskussion zu stoppen: Mein Argument ist einfach, dass der wissenschaftliche Konsens * königlich * vermasseln kann. Wenn Sie nicht glauben, dass dies passieren kann, lesen Sie einige der ursprünglichen Zitate prominenter Wissenschaftler über Wegeners Kontinentalverschiebung und Fritz Zwickys dunkle Materie zum Zeitpunkt der Entdeckung. Glauben Sie mir, Sie wollen das nicht lesen.
Das Neutrino-Ergebnis war ein ganz besonderer Fall. Es wurde sehr, sehr vorsichtig angekündigt, fast als "Ähm, Leute? Wir haben dieses irgendwie verrückte Ergebnis, das wir nicht erklären können. Wir haben es ewig versucht und wir haben Fehler gefunden und sie behoben und haben immer noch das Gleiche. Könnt ihr sehen Was haben wir falsch gemacht? " Mit dieser Sorgfalt in der experimentellen Arbeit, die möglicherweise Neutrinos zeigt, die schneller als Licht sind, lohnt es sich, darüber nachzudenken. Die meisten FTL-Ergebnisse sind nur ein Typ, der sagt: "Ja, aber was ist, wenn Einstein falsch liegt und ..." oder schlimmer noch: "Einstein sagt, dass dies unmöglich ist, aber wenn wir dies subtil neu definieren, dann ..."
@DavidRicherby Und weil sie gefragt haben, was sie falsch gemacht haben, sollten die Leute warten und Vorschläge senden, anstatt FTL-Papiere einzureichen. Die Verletzung der Relativitätstheorie ist ein * wirklich schreckliches * Problem. Alle Partikel haben eine "normale" Ruhemasse (m = 0 => v = c / m> 0 => v
Philip Gibbs
2014-03-25 16:28:41 UTC
view on stackexchange narkive permalink

Ungeachtet dessen, was die Leute sagen werden, ist es wahr, dass Zeitschriften Artikel mit Autorenprofilen ohne ordnungsgemäße Überprüfung ablehnen. Es ist schwer zu sagen, wie viele Papiere auf diese Weise abgelehnt werden, aber Elsevier sagt, dass sie 30 bis 50 Prozent der Papiere ohne Überprüfung aus anderen "technischen Gründen" ablehnen. Siehe auch dieses Papier darüber, wie Redakteure Zeit sparen können, indem sie sich Autorenattribute wie die Zugehörigkeit zu Ablehnungspapieren ansehen, ohne sie anzusehen.

Ich habe persönliche Erfahrungen damit, weil ich kürzlich machte bedeutende Fortschritte bei einem bekannten 100 Jahre alten offenen Problem, nachdem Experten auf diesem Gebiet gesagt hatten, dass die zukünftigen Fortschritte wahrscheinlich sehr langsam sein würden. Die Zeitschrift, in der ich das Papier eingereicht habe, um es abzulehnen, sobald ich bestätigt habe, dass ich keine Zugehörigkeit habe. Es gab keinen Gutachterbericht und sie gaben keinen bestimmten Grund an. Ich hatte alle technischen Anforderungen für die Einreichung erfüllt.

Ich wies sie jedoch darauf hin, dass gemäß dem Verhaltenskodex des Ausschusses für Veröffentlichungsethik an die sich die Zeitschrift zu halten behauptet: "Redaktionelle Entscheidungen sollten nicht von den Ursprüngen des Manuskripts beeinflusst werden" und "Zeitschriften sollten einen erklärten Mechanismus haben, mit dem Autoren gegen redaktionelle Entscheidungen Berufung einlegen können". Zu meiner Überraschung antworteten sie nach einer Verzögerung und sagten mir, dass sie es sich noch einmal ansehen würden.

Es ist wahr, dass es viele behauptete Lösungen von Problemen wie P gegen NP gibt, die auf einen Blick abgewiesen werden können . Dies kann geschehen, weil es gut verstandene Gründe gibt, warum diese Probleme schwierig sind, und eine Lösung erforderlich wäre, um dies zu beheben. Viele behaupteten Beweise für offene Probleme von Nicht-Akademikern gehen schnell in eine nicht standardisierte Sprache über, was es schwierig macht, überhaupt zu erklären, warum sie falsch liegen, so dass sie von der Community einfach ignoriert werden. Es liegt an den Autoren, sicherzustellen, dass sie ihre Ideen richtig kommunizieren.

Wenn Sie eine Lösung für ein offenes Problem haben, empfehle ich, diese an ein offenes Repository wie arXiv zu senden. Wenn Sie keinen Endorser bekommen können, verwenden Sie viXra oder figshare (vollständige Offenlegung: Ich bin viXra-Administrator). Achten Sie nicht auf negative Aussagen über viXra. Es dient lediglich dazu, Ihnen einen unabhängigen Zeitstempel und eine archivierte Kopie zu geben, auf die Sie verweisen können. Es wird nicht versucht, Ihre Arbeit in irgendeiner Weise zu überprüfen oder glaubwürdig zu machen. Das Letzte, was Sie tun sollten, ist, Zeitschriften einzureichen oder an Experten zu senden, ohne über eine verifizierte öffentliche Kopie zu verfügen. Wenn es sich tatsächlich um einen Durchbruch handelt, besteht ein echtes Plagiatsrisiko, das nur durch die Archivierung einer vorherigen Kopie abgewendet werden kann.

Ihr erster Absatz scheint darauf hinzudeuten, dass der Link zu einem Elsevier-Redakteur führt, der zugibt, dass er Autorenprofile verwendet, um Beiträge abzulehnen, aber es gibt keinen solchen Link (er listet absolut gültige technische Gründe für die Ablehnung eines Papiers auf). Beachten Sie auch, dass in der mathematischen Community das Setzen von etwas auf viXra Sie als weniger ernst markiert (ob dies der Fall sein sollte oder nicht). Daher würde ich niemandem raten, dort etwas hochzuladen, es sei denn, er hat wirklich keine anderen Optionen.
Mein erster Absatz sagt nicht, was Sie dachten. Sie lesen Dinge zwischen den Zeilen, die nicht da sind. Wenn Sie als nicht verbundener Autor einen Beweis für ein bekanntes Problem beanspruchen, werden Sie bereits Probleme haben, die ernst genommen werden, und es wird Ihnen sehr schwer fallen, sie auf arXiv hochzuladen. Selbst wenn Sie einen Endorser haben, wird arXiv für Tage bis zur Überprüfung durch den Moderator immer eine nicht verbundene Einreichung aufbewahren und diese dann möglicherweise ablehnen, um den Autor einem möglichen Plagiat auszusetzen. viXra wird ohne solche Moderation sehr schnell hochgeladen. figshare scheint auch schnell zu funktionieren, wenn Sie es vorziehen.
Mir ist völlig bewusst, was in der ersten Zeile steht. Ich erwähne Ihnen, was es für jemanden bedeuten könnte, der es zum ersten Mal liest.
Ich werde es ändern und versuchen, es klarer zu machen
Auf der Grundlage seines Profils listet PG seine Website als [Vixra Blog] (http://blog.vixra.com) auf und scheint mit der Website [Vixra] (http://blog.vixra.org/) verbunden zu sein, eine Alternative an arXiv, was in der Antwort nicht angegeben ist
Sie müssen die Antwort erneut lesen, in der "Vollständige Offenlegung: Ich bin viXra-Administrator" steht. Beachten Sie auch, dass es sich nicht um eine kommerzielle Website handelt. Wir müssen nicht für Geschäfte werben. Ich erwähne es zusammen mit einer Alternative nur, weil es ein relevanter Teil der Antwort ist.
* "Wenn es wirklich ein Durchbruch ist, besteht ein echtes Plagiatsrisiko" *. Dies ist die Art von Dingen, bei denen Menschen häufig als Trottel identifiziert werden - sie glauben, dass Redakteure und Rezensenten ihre Arbeit stehlen und sie gleichzeitig als ungültig ablehnen werden. Glauben Sie wirklich, dass dies ein Risiko ist, wenn Sie bei seriösen Zeitschriften einreichen, und sind Ihnen Fälle bekannt, in denen dies geschehen ist?
Ja, ich glaube, es besteht ein Risiko, und selbst wenn es ein kleines ist, sollten die Autoren Maßnahmen ergreifen, um dies zu vermeiden. Mir sind keine identifizierbaren Fälle bekannt, aber das schließt nicht die Möglichkeit und das Risiko aus, dass dies geschieht. Das Plagiat in der Wissenschaft wird so offensichtlich, dass Sie jetzt Papiere finden können, die anscheinend fast vollständig Wort für Wort dreimal vollständig kopiert wurden, siehe http://academia.stackexchange.com/questions/18512/how-can-i- Finden Sie den Originalartikel einer oft plagiierten Arbeit. Es wäre naiv zu glauben, dass dies nicht die Spitze des Eisbergs ist.
Schlimmer noch: Beiträge werden aufgrund von Autorenprofilen * akzeptiert *. Das heißt, wenn Sie 20 Artikel aus einer ständig wachsenden Anzahl von Einreichungen auswählen müssen (Leute in einigen Bereichen sind gekommen, um zu veröffentlichen, wie sich Kaninchen fortpflanzen), kann ich verstehen, warum Sie 50 Artikel auswählen würden, von denen Sie annehmen, dass sie von * einigen * gut sind. Kriterium und lassen Sie nur sie überprüfen. Verstehe, nicht wie.
Ja, es ist verständlich, wenn Redakteure Zeit sparen möchten, aber es wäre unehrlich, dies zu tun und dann zu behaupten, dass sie den COPE-Verhaltenskodex einhalten.
@PhilipGibbs In dem von Ihnen verlinkten Beitrag geht es nicht um Plagiate von Rezensenten oder Herausgebern seriöser Zeitschriften, die das Material abgelehnt und dann als ihr eigenes weitergegeben haben. Kennen Sie also wieder Fälle, in denen dies geschehen ist?
David, das Risiko, dass es passiert, besteht unabhängig davon, ob es jemals passiert ist oder dokumentiert wurde. Der Rat, den ich jedem gebe, der eine Entdeckung macht, ist, diese Chance nicht zu nutzen, wenn er die Priorität seiner Arbeit schätzt.
@PhilipGibbs Natürlich. Ebenso besteht das Risiko, dass Außerirdische Ihre Arbeit stehlen und daraus einen Todesstrahl erzeugen, der uns alle tötet, unabhängig davon, dass dies niemals geschehen ist. Die Frage ist, ob diese Risiken groß genug sind, um sich Sorgen zu machen. Das Ausmaß der Risiken ist * nicht * unabhängig davon, ob sie jemals aufgetreten sind oder nicht.
Ich habe einige Fallakten gefunden, in denen Gutachter beschuldigt wurden, die Arbeit, die sie prüfen sollten, plagiiert zu haben. In beiden Fällen kam man zu dem Schluss, dass es nur ein Versäumnis war, einen Interessenkonflikt zu erklären, wenn sie an ähnlichen Forschungsarbeiten arbeiteten. In einem Fall (http://publicationethics.org/case/parallels-between-unpublished-manuscript-and-published-article-other-authors) wurde das Papier vom Rezensenten abgelehnt und im anderen Fall (http: // publicationsethics). org / case / reviewerauthor-Konfliktinteresse) er hatte die Überprüfung einfach nicht durchgeführt.
David, wenn du denkst, dass die Risiken nicht hoch sind, ist das in Ordnung für dich. Ich würde das Risiko nicht eingehen.
@jwg Dieses Risiko besteht und es hilft nicht, es lächerlich zu machen. Siehe [Jocelyn Bell] (https://en.wikipedia.org/wiki/Jocelyn_Bell) und [Rosalind Franklin] (https://en.wikipedia.org/wiki/Rosalind_Franklin). Als Student fand unsere Gruppe heraus, dass ein Assistent die Daten eines anderen Assistenten ohne Erlaubnis verwendete. Mark Chu-Carroll von "Good Math, Bad Math" hatte eine schlechte Begegnung, als er mit einer anderen Person über eine neue Idee sprach und später herausfand, dass diese Person ohne Anerkennung ein Papier mit dieser Idee herausbrachte.
@ThorstenS., Wie David Richerby betonte, ist die Auflistung verschiedener Plagiatsfälle nicht gleichbedeutend mit der Angabe eines Einzelfalls, in dem Gutachter oder Redakteure eingereichtes Material nur zur Plagiierung abgelehnt haben. Ich habe das nicht „lächerlich gemacht“ - ich habe darauf hingewiesen, ich hoffe hilfreich, dass diese Art von Behauptung genau das ist, wonach die Leute Ausschau halten, um Kurbeln zu erkennen. Wenn Sie ernsthafte Arbeit einreichen, tun Sie sich selbst einen schlechten Dienst, indem Sie launisch erscheinen. Wenn Ihre eingereichten Arbeiten jedoch wirklich plagiiert wurden, sollten Sie sich darüber aufregen.
@jwg Nun, los geht's: Die amerikanischen Autoren Vijay R.Soman und Philip Felig von der University Yale haben * genau * das getan: Ein Papier von einer NIH-Gruppe abzulehnen und ihre Daten für ihren eigenen Zweck zu stehlen. Die deutschen Krebsprofessoren Herrmann und Brach lehnten einen Zuschusswunsch einer niederländischen Gruppe ab, übersetzten die ursprünglichen Argumente aus dem Niederländischen ins Deutsche und baten damit um einen eigenen Zuschuss.
@ThorstenS. Ich finde es eigentlich sehr beruhigend, dass das beste Beispiel eines von vor mehr als 30 Jahren zu sein scheint, und das nicht einmal der Plagiat des Rezensenten ist (vielmehr hat er einen möglichen Interessenkonflikt nicht gemeldet und das zu überprüfende Papier einem gezeigt untergeordnet, wer es dann plagiiert hat).
@TobiasKildetoft Ähm, Herrmann / Brach stammt aus dem Jahr 2003. Im Voraus: Ich denke, dass die meisten Wissenschaftler ehrlich sind. Abgesehen von der Tatsache, dass der Torpfosten jetzt verschoben wird (10/100/1000 Fälle?), Sollte ich erwähnen, dass Universitäten verklagt werden könnten, um Zuschüsse vollständig zurückzuzahlen, wenn Betrug entdeckt wird. Eine weitere interessante Tatsache könnte sein, dass Betrug (wie Jan-Hendrik Schön) meist von anonymen Hinweisen erfasst wird, die möglicherweise einen Grund dafür haben, dass Reiner Protsch die Karrieren mehrerer Personen zerstört hat, die versucht haben, andere Wissenschaftler zu warnen.
@ThorstenS. Richtig, aber das war kein Rezensent eines neuen Ergebnisses, das war eine etwas andere Situation.
Ich hatte sowohl mit Kurbeln als auch mit echten Plagiatsfällen zu tun - beide existieren.Die Tatsache, dass Sie mit etwas Glück keinen Verkehrsunfall haben, bedeutet nicht, dass Sie nachlässig sein sollten.Ich kann mit jemandem sympathisieren, der glaubt, ein bahnbrechendes Ergebnis zu erzielen, um seine Priorität zu schützen (siehe auch Perelman vs. Chern, wo selbst die Tatsache, dass er als kompetent angesehen wurde und das relevante Papier früh genug da draußen war, einen Prioritätsstreit nicht verhinderte).Nicht verbundene oder schwach verbundene Forscher sind besonders gefährdet.Ich habe meine Zweifel an Kurbeln, aber lassen Sie uns die Plagiatsgefahr nicht abtun.
@CaptainEmacs Entschuldigung, es scheint keinen Perelman gegen Chern (der 2004 starb) zu geben, und der Grund für Perelmans Abgeschiedenheit scheint ungerechtfertigt zu sein (ich glaube den Berichten über The New Yorker nicht ganz).
Michael Wehar
2015-02-05 11:51:26 UTC
view on stackexchange narkive permalink

Aus meiner Erfahrung als Doktorand scheint es, dass Forscher, die sich für ihre Arbeit begeistern, ungefähr dasselbe tun, unabhängig von ihrer vermuteten Bedeutung .

Hier ist Was zu tun ist:

(1) Schreiben Sie Ihre Arbeit so gut wie möglich auf und besprechen Sie die Ideen und Konzepte mit Ihren Freunden, Mitarbeitern, Mitarbeitern usw.

(2) Bewerben Sie Ihre Arbeit bei anderen Forschungen, Freunden, Mitarbeitern usw. Aber oft sind die Leute beschäftigt, also halten Sie Ihre Erwartungen niedrig und die Höflichkeit hoch.

(3) Finden Sie Ihr Publikum heraus und suchen Sie die geeignete Zeitschriften, Workshops und Konferenzen, an die Sie sich wenden können.

(4) Wenn Sie vor dem Einreichen Fragen oder Bedenken haben, wenden Sie sich an eine mit der Konferenz verbundene Person. Ich habe positive Erfahrungen damit gemacht, aber manchmal werden Sie ignoriert.

(5) Seien Sie zur Ablehnung bereit, da Sie mit größerer Wahrscheinlichkeit abgelehnt werden (auch wenn Ihre Ergebnisse wichtig sind, insbesondere) wenn sie wichtig sind).

Anmerkungen:

(a) Ich denke gern, dass die Qualität der Bewertungen mit der Klarheit des Papiers zusammenhängt. Aber ich habe in der Vergangenheit einige fragwürdige Kritiken erhalten, die wie verstreute Worte erscheinen, die möglicherweise mit meiner Arbeit zusammenhängen oder nicht. Halten Sie einfach keinen Groll und versuchen Sie es gerne erneut.

(b) Wenn Ihnen etwas wichtig ist und Sie finanziell in der Lage sind, ist es in Ordnung, ein- oder zweimal abgelehnt zu werden, solange Sie versuchen, sich zu verbessern Ihre Präsentation und kommunizieren Sie weiter mit anderen.

(c) Sie möchten nicht vom Erdboden verschwinden und wie ein Einsiedler leben. Das wird niemandem helfen und besonders Ihnen nicht.

(d) Seien Sie schließlich aufgeschlossen. Menschen machen Fehler und manchmal ist das, was den Fehler verursacht hat, bedeutungsvoll.

Laurent Duval
2016-09-24 17:38:08 UTC
view on stackexchange narkive permalink

Einige sehr wichtige Artikel wurden zuerst abgelehnt, andere wurden nicht einmal veröffentlicht.

Wenn Sie sich über Ihr Ergebnis hinreichend sicher sind und einen Termin für Ihre Entdeckung festlegen möchten und keine Angst davor haben Manchmal braucht es Zeit, um die Leute von der Richtigkeit zu demonstrieren und zu überzeugen. Stellen Sie Ihr Papier online in einem offenen Archiv, während Sie an einer hoffentlich veröffentlichten Version arbeiten.

Einige Leser entdecken möglicherweise einen Fehler und helfen Ihnen möglicherweise beim Veröffentlichen.

Tom Au
2016-09-12 09:26:34 UTC
view on stackexchange narkive permalink

Erstens geschieht "das Lösen berühmter offener Probleme" nicht im luftleeren Raum. Es muss einen guten Grund geben, warum die Lösung Ihnen einfiel, nachdem Sie sich vielen anderen entzogen hatten.

Ein möglicher Grund ist, dass Sie ein Experte für eine neue Technik oder Analysemethode sind. Dann besteht der Trick darin, sich als Experte auf diesem "neuen" Gebiet zu etablieren. Sobald Sie dies getan haben, ist es einfacher zu behaupten, dass Ihre Beherrschung dieses einen Bereichs es Ihnen ermöglicht hat, das "ungelöste" Problem zu lösen (vorausgesetzt, Sie können die Relevanz Ihres Fachgebiets nachweisen). Wenn Sie beispielsweise ein Pionier der subatomaren Physik wären, der herausfindet, dass die Newtonsche Physik im subatomaren Bereich nicht funktioniert, hängt die Akzeptanz Ihres "Beweises" von der Akzeptanz der Menschen als subatomarer Experte ab.

Die andere Sache ist, wenn Sie wirklich eine neue Lösung für ein Problem gefunden haben, bedeutet dies, dass vieles, was derzeit in Bezug auf dieses Problem auf dem Gebiet geschrieben wird, falsch ist oder zumindest überdacht werden muss. Der Weg, sich zu beweisen, besteht darin, Anwendungen auf einer niedrigen Ebene zu identifizieren und auf zunehmend höhere Ebenen aufzusteigen, die jetzt durch Ihre Entdeckung "ungültig" werden. Wenn Sie beweisen können, dass ein ganzer "Strom" von Ideen überdacht werden muss, und dann Ihre Entdeckung als "gemeinsame" Lösung präsentieren, werden die Leute Sie viel ernster nehmen. Ein Beispiel war, als Leute herausfanden, dass man ein neues System "nichteuklidischer" Geometrie erstellen kann, indem man nur einige Annahmen ändert.

Ooker
2019-03-23 16:56:23 UTC
view on stackexchange narkive permalink

Bereite dich auf deine Furchtlosigkeit vor. Sie müssen es häufig verwenden.


Nehmen wir an, unser Ziel ist es, Kavehs Liste zu erreichen. Vorausgesetzt, er hat Erfahrung in der akademischen Forschung, dann ist jede dieser Aussagen gleichwertig:

  • Seine Priorität ist nicht wirklich, dass seine Arbeit akzeptiert wird, sondern dass er nicht missverstanden wird (die eigentliche Prämisse der.) Bei der Frage geht es um produktives Feedback.)
  • Für ihn gibt sie nur das Offensichtliche an (den oberen Teil der Liste) und sagt ihm, er solle Dinge tun, die er bereits tut (der untere Teil).
  • Sie braucht ihn, um die Liste zu erstellen, aber er kann das nicht, wenn sie ihn beschuldigt, das getan zu haben, was er nicht getan hat.
  • Das Problem, sie davon zu überzeugen, dass er keine Kurbel ist, reduziert sich auf Die emotionalen und missverständlichen Probleme

Ich denke, dass in allen Diskussionen über Kurbeln die überwältigende Annahme ist, dass er Hybris ist. Dies ist leider einseitig, da die tatsächlichen Emotionen Schuld und Frustration sein sollten. Er ist schuldig, weil er sich Sorgen macht, er sei Hybris, und frustriert, weil sie falsch eingeschätzt hat, er sei Hybris. Bei allem Respekt denke ich, dass den meisten Experten die notwendige Erfahrung fehlt, um einen effizienten Rat zu geben.

Da diese Antwort nicht versucht, die Probleme zu beantworten, die die meisten etablierten Experten ansprechen möchten, ist sie im Grunde genommen Eine Frame Challenge Antwort. sup>

Der Rest der Antwort ist nur eine Ausarbeitung des Umgangs mit emotionalen Reaktionen und Missverständnissen in jedem Einzelfall. Hier ist das Inhaltsverzeichnis für diese Antwort:

  • Umgang mit Ihren Emotionen
  • Umgang mit ihren Reaktionen
  • Andere Probleme
  • Für diejenigen, die helfen möchten
  • Links

Sie können auch eine Antwort auf eine allgemeine Frage lesen ( welche Herausforderungen während des Projekts auftreten können, nicht nur, wie Sie die Experten überzeugen können). Ich gehe davon aus, dass Ihre Plattform, um Feedback zu Ihrer Arbeit zu erhalten, Reddit ist, aber es ist keine große Sache.

Umgang mit Ihren Emotionen

Zunächst gibt es zwei Dinge:

  • Erkenntnistheoretisch scheint es, dass Sie sich der Lösung bewusst sein müssen, um ein schwieriges Problem zu lösen. a priori
    Erkenntnistheorie ist a Feld in der Philosophie über die Natur des Wissens und wie wir es erwerben. Ich habe jedoch nicht viel darüber gelesen. Sup>
  • Wenn Sie sich Ihrer selbst bewusst sind, werden diese selbstbewussten Emotionen psychologisch hervorrufen: Stolz, Scham, Schuld und Verlegenheit

Weil Forschung bedeutet, zu beweisen, dass Sie falsch liegen, bevor Sie beweisen, dass Sie Recht haben. Eine natürliche Sache, wenn Sie glauben, etwas zu wissen, ist Skepsis gegenüber sich selbst. Zusammen mit dem spezifischen Problem, mit dem Sie arbeiten, werden Sie diese selbstbewussten Fragen stellen:

  • Behaupte ich etwas, das ich noch nicht bewiesen habe?
  • Behaupte ich, dass ich es besser weiß als die Experten, wenn mir die formale Ausbildung fehlt?

Da die Antworten auf diese Fragen ja sein sollten, rufen sie selbstbewusste Emotionen hervor und treiben Sie von der Eine und einzige Sache, auf die Sie sich konzentrieren sollten: das Problem, an dem Sie arbeiten. Sie werden in eine Rückkopplungsschleife eintreten, in der Sie sich selbst befragen: Die Antwort auf diese Fragen ist der Grund, warum Sie sich zuerst selbst befragen. Die eigentliche Antwort liegt in der Arbeit, die Sie erforschen, und nicht darin, ob Sie gesund sind oder nicht.

Um mit Fantasie (einer Art Stolz) umzugehen, versuchen Sie sich vorzustellen, wie ein Leben einer berühmten Person tatsächlich aussehen würde. Für jeden Erfolg, von dem Sie träumen, gibt es Menschen, die ihn bereits haben. Sagen Sie also, Sie träumen vom Nobelpreis, stellen Sie sich folgende Frage: Wie hat Einstein mit seinem Ruhm und Geld gelebt? Wenn Sie feststellen, dass tatsächlich alle berühmten Personen sich darüber ärgern, berühmt zu sein, wird Ihre Fantasie unterbrochen. Indem Sie sich in ihre Lage versetzen können, können Sie sich von Ihren Emotionen lösen und zur Realität zurückkehren. Wenn Sie sich zu diesem Zeitpunkt immer noch fragen, was Sie tun werden, wenn Sie berühmt werden, möchten Sie nur einen dunklen und ruhigen Ort, an dem Sie traumlos schlafen können.

Bitte beachten Sie, dass Sie dies tun können habe Messias-Komplex. Sie haben vielleicht keine grandiose Täuschung, aber der Gedanke, dass (1) nur Sie eine Lösung anbieten können, die niemand zu sehen scheint, und (2) um diese Lösung zu liefern, müssen Sie die Skepsis überwinden, unabhängig davon, wie hart sie ist. kann diesen Komplex entwickeln. Möglicherweise werden Sie auch beschuldigt, Gasbeleuchtung durchgeführt zu haben. All dies wird die Schuld an sich selbst noch stärker machen.

Um sie mit echten psychologischen Problemen zu unterscheiden, deren Wurzel in der Unsicherheit liegt, werde ich sie als "intellektuellen Messias-Komplex" und "intellektuelles Gaslicht" bezeichnen. . sup>

Weil die Angst, dass Sie eine Wahnvorstellung haben, nur dann anhält, wenn Sie tatsächlich sehen, dass Sie noch Fantasie haben. Wenn Sie sie also erfolgreich geschnitten haben, verschwindet die Angst selbst. Sie werden sich wegen Ihrer Arbeit nicht mehr schämen, schuldig oder in Verlegenheit bringen.

Im Allgemeinen sind die Emotionen nur dann intensiv, wenn sich Ihre Forschung noch in der vagen Phase befindet. Je mehr Wissen Sie erwerben, desto seltener treten sie auf.

Umgang mit ihren Reaktionen

Nach meiner Erfahrung gibt es diese Art von unproduktiver Reaktion: Markierung und Sarkasmus. P. >

Beschriftung

Wenn sie sagen, dass Sie eine Kurbel sind, geben Sie ihnen einfach diesen Crackpot-Index, listen Sie alle Punkte auf, die für Sie zutreffen könnten, und erklären Sie, wie sie falsch sind. Wenn sie Sie beschuldigen, arrogant zu sein, sagen Sie ihnen, dass es etwas anderes ist, selbstbewusst mit einem Lächeln in einen Sturm zu gehen, als nach Aufmerksamkeit zu suchen. Wenn sie denken, dass Sie verrückt sind (mit einem Lächeln in einen Sturm zu gehen, ist offensichtlich verrückt), wäre es viel einfacher, wenn Sie das Gespräch von Angesicht zu Angesicht führen können. Nur wenn Sie sehen, wie bestimmt Ihre Augen sind, aber Ihr Geist überhaupt nicht geschlossen ist, können sie davon ausgehen, dass Sie es nicht sind. Schäme dich nicht, wenn du ihnen erzählst, wie du dich auf das Projekt vorbereitet hast.

Lass uns mehr über Wahnsinn diskutieren. Ich denke, es ist am besten, ihnen zu helfen, zu dem Schluss zu kommen, dass Sie verrückt sind, weil es nicht länger ein Etikett für Ihr Verhalten ist, sondern ein Etikett für Ihre Rationalität. Wenn Sie dieses Stadium erreichen, haben sie eine starke Motivation, das Gespräch fortzusetzen, und Ihre Beweise werden sorgfältig angehört. Wenn Sie eine kognitive Dissonanz in ihrem Kopf erzeugen können, ist ihre Musterübereinstimmung stumm und sie sind nicht mehr in ihrer Perspektive eingeschlossen.

Reagieren Sie auf ihre Kennzeichnung (Kurbel, arrogant, verrückt, dumm) , hoch, Voreingenommenheit, Spam, Wortsalat, nicht einmal falsch, woowoo, Pseudowissenschaft, Zeitverschwendung usw.), indem Sie die Definition des Wortes angeben und zeigen, warum es für Sie nicht gilt. Halten Sie eine Notiz mit allen vorbereiteten Antworten für jedes Etikett bereit, damit Sie sich nicht auf Ihr schlechtes Gedächtnis verlassen müssen. Um mit Skepsis umzugehen, müssen Sie sofort eine perfekte Wortkombination bilden, und Ihr Gehirn wird Sie stürzen. (Weitere Details später.)

Sarkasmus

Wenn sie nur ein paar Witze machen, kann es wahr sein, dass es tatsächlich lustig ist. In diesem Fall ist es vielleicht am besten, den Witz fortzusetzen . Wenn sie sehen, wie Sie ihre Witze tatsächlich genießen, werden sie sehen, dass es Sie überhaupt nicht betrifft. Wenn Sie beispielsweise eine dicke Person sind, werden Sie durch einen Scherz über Ihre Fettleibigkeit erkennen, dass Sie sich Ihres Mangels bewusst sind, und es ist nur so, dass Sie noch keine Zeit haben, es zu lösen. Wenn Sie ein Blinder sind, wird ein Witz, der Ihre Behinderung betrifft, die Nicht-Blinden erstaunlich überraschen. Sie werden dich jetzt als unbesiegbar wahrnehmen.

Wenn es wirklich weh tut und dir nichts Kluges einfällt, ist es vielleicht am besten, sie daran zu erinnern, dass sie sich an einem Ort befinden, an dem Spott nicht akzeptabel ist . Wie eine Show sehen sie dich buchstäblich als den arroganten, stolzen Kerl, der unterrichtet werden muss. Notieren Sie sich Links zu Comedic-Shows, bei denen es darum geht, sich über andere lustig zu machen, wie WWE-Shows, Videos zur Higaniga-Verschwörungstheorie oder verspottete Subreddits, und zeigen Sie ihnen, wie lustig ihre Aktionen sind. Zum Beispiel können Sie sagen:

Wow, r / Buddhismus wird jetzt r / WWE?

Oder Sie können eine Behörde einladen, sich damit zu befassen:

Ich halte dieses Verhalten nicht für angemessen. Ich werde die Mods melden / Ich glaube, andere finden das auch nicht angemessen.

Kurz gesagt, seien Sie schlau im Umgang mit Ad Hominem. Das Problem ist, dass Sie nicht klug sein können, wenn Ihr Geist von Wut getrübt ist. Sie müssen sich also auf clevere Momente vorbereiten. Wenn Sie dies tun können, wird ihre nächste Antwort wahrscheinlich wieder produktiv sein.

Aber interagieren Sie nicht mit ihnen, weil Sie sich missverstanden fühlen, sondern wegen des Wissens, das sie haben. Versuchen Sie, Fragen zu Ihrer Identität / Ihrem Selbst (z. B. "Bin ich Kurbel?") In Fragen zur Definition umzuwandeln (z. B. "Was ist Kurbel?"). Zeigen Sie, wie Sie einen starren Wissensbestand über das besitzen, was sie zu vermitteln versuchen, mit Sätzen wie "psychologisch gesehen", "was Sie sprechen, heißt ______ in der Philosophie", "im Bereich der informellen Logik" usw. Sie sind relevante interessante akademische Bereiche, von denen sie nichts wissen. Wenn Sie der Meinung sind, dass sie Kenntnisse über das haben, was Sie brauchen, konzentrieren Sie sich nur darauf und ignorieren Sie jede unangemessene Kritik an Ihnen.

Wenn Sie alle emotionalen Probleme gelöst haben, sind 90% der Gespräche jetzt produktiv. Aber es wird noch andere Probleme geben.

Andere Probleme

Gedenkproblem

Möglicherweise haben Sie so viel Zeit damit verschwendet, über Crackpotismus, Verschwörungstheorien, Täuschungen usw. Zu lesen Sie haben vielleicht so viel Zeit mit Gesprächen verschwendet, dass die Leute zunächst nicht mit Ihnen übereinstimmen, aber nach einigen Gesprächen stellt sich heraus, dass sie nicht wirklich mit dem übereinstimmen, was Sie tatsächlich meinen, und all dies ist nur ein Missverständnis. Beachten Sie alle Beweise dafür, dass Sie sich auf dem richtigen Weg fühlen. Lassen Sie Ihre Nachforschungen über Ihre geistige Gesundheit nicht los.

Das Problem, sich nicht erklären zu können, liegt an implizitem Wissen und der Zungenspitze Phänomen. Wenn Sie einen sehr starken Beweis dafür haben, dass Sie keine Kurbel sind, dann ist nach ungefähr einer Woche nur noch das Gefühl übrig, dass Sie es nicht sind. Fehlen die Beweise, werden Sie erneut in die Schleife gezogen. Sie müssen sofort eine perfekte Kombination von Wörtern erstellen, um daraus herauszukommen.

Missverständnis

Selbst wenn ein Wunder geschieht (sie verbringen ihre Zeit und Energie damit, zu analysieren, was Sie sagen), wird es ein sehr seltsames Phänomen geben, das Sie einfach nicht verstehen können: Sie haben immer das Gefühl, dass sie implizit mit dem übereinstimmen, was Sie sagen, aber Sie Zwei können sich nicht auf einen Konsens einigen und werden daher nur kreisförmig.

Ich denke, alle Konflikte in der Welt sind auf das Problem der Priorität zurückzuführen. Normalerweise sieht die Situation so aus: Person 1 kann sehen, dass Problem A wichtiger ist als Problem B, und Person 2 sieht, dass B wichtiger ist als A. Das Problem ist, dass meistens sowohl A als auch B gelöst werden müssen zusammen, oder keiner von ihnen kann erreicht werden. Aber weil beide darauf bestehen, dass ihr Punkt wichtiger ist, werden beide den Punkt verfehlen und sich gegenseitig überholen. Beide werden das Gefühl haben, dass das Gespräch unproduktiv ist, und früher oder später wird man es fallen lassen.

Wenn dies geschieht, ist es nur ein Blinder, der einen Blinden führt, oder schlimmer noch, ein blinder Kampf. Die Untersuchung der Natur dieses Phänomens und des Umgangs damit ist mein Forschungsinteresse. Mein Rat, mit Missverständnissen umzugehen, ist, Negation zu verwenden, keine Erklärung.

Egozentrismus

Egozentrismus bedeutet nicht, egoistisch zu sein oder Ich habe ein riesiges Ego, aber es geht mir nicht darum, deinen Geist von dem anderer zu unterscheiden. Wenn Sie etwas Interessantes finden, weisen Sie automatisch zu, dass andere es ebenfalls interessant finden, und sind verwirrt, wenn es ihnen tatsächlich egal ist. Wie auch immer Sie sich daran erinnern, diese Tendenz wird immer noch aktiviert.

Gehen Sie nicht davon aus, dass sie davon ausgehen, dass ihr Feedback falsch ist. Sie mögen aufgeschlossen sein, und das sind sie auch, aber in der Praxis können Sie beide nicht. Wenn Sie davon ausgehen, dass sie neugierig sind, warum sie falsch liegen (eine Art Vertrauen), neigen Sie dazu, Erklärungen abzugeben. Aber tatsächlich sehen sie nur, dass Sie defensiv sind. Sie beschuldigen Sie für Dinge, die Sie noch nie getan haben, und lassen das Gespräch dann entweder passiv fallen oder blockieren Sie aktiv von weiteren Erklärungen. Ich bezeichne dies als "intellektuelle stille Behandlung".

Ich suche Feedback

Wenn eine Idee auftaucht und Sie sich wunderbar fühlen, wird Ihnen Ihr Bauch das immer noch sagen Es gibt viel mehr Felder, die Sie sorgfältig lesen müssen. Obwohl es Ihnen nichts ausmacht, mehr Aufwand für die Recherche aufzuwenden, möchten Sie nur um Feedback bitten, da dies viel effizienter wäre. Ihr Egozentrismus geht davon aus, dass die Leute auch das bekommen, was Sie bekommen. Wenn Sie das Projekt so schnell wie möglich beenden möchten, um zu anderen wichtigen Dingen überzugehen, ist Ihr Drang, es zu teilen, höher. Aber hör auf! Wenn Sie es jetzt veröffentlichen, erhalten Sie nur harte, unvernünftige Kritiken. Hören Sie auf Ihren Bauch und lesen Sie alle Felder, die Sie zuerst benötigen. Gute Fragen kommen nur, wenn sich Ihr Geist in der Phase der Unwissenheit befindet. Dies ist das Ergebnis des Verständnisses des Feldes, wie es ist.

Allerdings werden Sie irgendwann feststellen, dass es keinen Grund gibt, Fragen zu stellen und Sie möchten nur, dass sie Ihre Arbeit so lesen, wie sie ist. Wenn Sie über die erforderlichen Anforderungen verfügen (Literaturübersicht, Inline-Zitierung, Methodik usw.), können Sie sich jetzt an eine akademische Zeitschrift wenden, und Sie müssen keine Angst mehr haben, als Kurbel missverstanden zu werden.

Feedback erhalten

Wenn Ihr erstes und wichtigstes Publikum Ihrer Arbeit das beliebte Publikum ist und Ihr Schreibstil notwendigerweise unakademisch ist, wird die Situation kompliziert, da Ihr wichtigstes Publikum das akademische ist. (Ja, manchmal ist das Wichtigste nicht das, das Sie priorisieren sollten - siehe die Eisenhower-Methode.) Da Ihre Arbeit zwei verschiedene Arten von Publikum bedienen muss, die unterschiedliche Hintergründe, Kenntnisse und Erwartungen haben, werden Sie beim Empfang gemischte Gefühle haben Feedback:

  • Das beliebte Publikum kann kein nützliches Feedback geben, aber seine Aufregung zeigt an, dass Sie ein großes Problem angesprochen haben, nach dem es sucht.
  • Fachkundigere Leser oder sogar Akademiker aus der Ferne Felder können nützliches Wissen vermitteln und die Rolle der ersten Gatekeeper spielen. Aber wenn sie Ihnen viel Glück sagen, wissen Sie, dass sie Ihnen nicht mehr helfen können.
  • Akademiker aus relevanten Bereichen werden das Gefühl haben, dass es vage oder verwaschen ist, weil sie erwarten, dass Ihre Arbeit wirklich in akademischer Form präsentiert wird. Wenn Sie jedoch ausdrücklich sagen, dass dies nur die Phase ist, in der Sie erfassen, was Sie im Kopf haben, dreht sich ihre Einstellung um 180 Grad.

Lassen Sie uns mehr über den letzten Punkt sprechen. In Ihren Augen ist es nicht wichtig, keine literarische Rezension, Methodik oder keinen Datensatz zu haben, da Sie bereits akzeptieren, dass Sie keinen haben. Daher müssen Sie ausdrücklich sagen, dass der Artikel nur eine flüchtige Recherche ist, um eine Roadmap für Ihre Studie zu skizzieren und alle Ihre Mängel so gut wie möglich aufzulisten. Ohne diesen Teil wird ihre Erwartung ihren Verstand sperren, und jede Ihrer Erklärungen wird von nun an als defensiv empfunden.

Für diejenigen, die helfen möchten

Hier ist mein Rat für diejenigen die helfen wollen. Hoffentlich kann es unnötigen Aufwand reduzieren und Ihnen maximales Glück bringen:

  • Gehen Sie immer davon aus, dass die Person, mit der Sie sprechen, etwas Interessantes hat, das Sie lernen können
  • Seien Sie sich bewusst, dass sie zwar nicht wissen, wovon sie sprechen, Sie aber möglicherweise auch nicht wissen, was sie wirklich vermitteln möchten.
  • Bestätigen Sie ihre korrekten Beobachtungen, bevor Sie Ihre Bedenken ansprechen
  • Verwenden Sie sokratische Fragen

Fühlen Sie sich nicht bedroht, wenn sie Anzeichen von Verschrobenheit zeigen. Die Tatsache, dass sie akzeptieren, als Kurbel gekennzeichnet zu werden, zeigt, dass sie etwas Wichtigeres zu tun haben. Wie Sie sind sie rationale Wesen und haben bereits vor Beginn des Projekts eine Kosten-Nutzen-Analyse durchgeführt. Wenn Sie ihnen Ratschläge geben möchten, möchten Sie sie möglicherweise folgendermaßen gestalten:

  • Ich glaube nicht, dass es eine andere Option für X gibt. / Der beste Weg, dies zu tun, ist Y. Aber Da es so aussieht, als ob Sie das auch wissen, können Sie erklären, warum Sie nicht glauben, dass es zu Ihnen passt?
  • Ich glaube nicht, dass Sie das Konzept Z so verstehen, wie es ist. Meiner Meinung nach handelt es sich bei Z um a, b oder c, und vielleicht ist c näher an dem, was Sie meinen. Ist das richtig?

Vielleicht möchten Sie etwas über konzeptionelle Metaphern lernen, wenn Sie wissen möchten, warum ernsthafte Ideen manchmal unglaublich verrückt sind. Ich empfehle das Buch Metaphern, nach denen wir leben von Lakoff und Johnson.

Links

Kletische hat einige gute Artikel dazu:

Sie können auch meine Forschung lesen: Eine Theorie der Perspektive. Es werden verschiedene Dinge besprochen, von denen zwei intellektueller Verrat und kalter Blick sind, die für diese Antwort relevant sind.

Was für eine absurd lange Antwort.Und doch scheint es kaum etwas Nützliches für das OP zu enthalten, da es größtenteils bedeutungslos ist.
Die anderen Antworten gehen davon aus, dass der Forscher gründlich kühl bleiben kann und die Experten die ersten Ideen nicht ein einziges Mal verspotten, sondern entweder entweder Feedback geben oder schweigen.Das ist nicht der Fall.Können Sie nach Möglichkeit erläutern, warum die ersten beiden Abschnitte bedeutungslos sind?
Denken Sie darüber nach, vielleicht nur die ersten beiden Abschnitte, die in direktem Zusammenhang mit der Frage stehen (wie man überzeugt).Bei den anderen Punkten handelt es sich eher um einen verallgemeinerten Punkt (welchen Herausforderungen man sich stellen kann).Ich werde sie dann entfernen
Dies beantwortet die Frage nicht.Diese Antwort beschreibt, wie OP mit ihrer eigenen mentalen / emotionalen Gesundheit umgehen soll (was wichtig ist, aber die angegebenen Ratschläge sind nicht korrekt), und hält der Forschungsgemeinschaft Vorträge darüber, wie sie Außenstehende behandeln sollen (die an Windmühlen kippen), von denen keine AdresseOPs Frage, wie sie andere Menschen am besten davon überzeugen können, dass ihre Lösung legitim ist.
_Wenn sie sagen, dass Sie eine Kurbel sind, geben Sie ihnen einfach diesen Crackpot-Index, listen Sie alle Punkte auf, die für Sie zutreffen könnten, und erklären Sie, wie sie falsch sindKurbel, die ich je gelesen habe.
@JeffE Es stellt sich heraus, dass diese Antwort eine [Frame Challenge] (https://interpersonal.meta.stackexchange.com/q/2511/86) Antwort ist, bei der eine scheinbar nicht zum Thema gehörende Beratung erforderlich ist.Ich füge die Erklärung dazu oben hinzu. Kannst du sie überprüfen?
Entschuldige Nein.Ich würde die Rahmenherausforderung für angemessen halten, wenn OP gefragt hätte: "Ich wurde als Kurbel bezeichnet. Wie kann ich die Leute davon überzeugen, dass ich es nicht bin?"oder "Wie gehe ich mit Ad-Hominem-Angriffen auf meine Forschung um?", aber das ist nicht das, was sie gefragt haben.(Und der Rat, den Sie anbieten, ist immer noch nicht verfügbar.)
@JeffE Was ist der Unterschied zwischen Ihrer ersten Beispielfrage und der Titelfrage?Und auf jeden Fall, welche Art von Frage halten Sie für angemessen und gut?
Der Unterschied ist, dass OP nicht sagt, dass er als Kurbel bezeichnet oder in irgendeiner Weise herabgesetzt wurde.Sie versuchen, solche Angriffe in erster Linie zu verhindern, nicht von ihnen zu erholen.
@JeffE Der Versuch, dies zu verhindern, führt zur [sich selbst erfüllenden Prophezeiung] (https://en.wikipedia.org/wiki/Self-fulfilling_prophecy).Wenn ein Neuling behauptet, er habe eine große Idee, dann muss er wegen seines Mangels an Jargon Analogie verwenden, um sie auszudrücken.Dies wird die Idee für die Experten so absurd und lustig machen, dass er früher oder später ausgelacht wird.Aus diesem Grund ist es besser, einen Ad-Hominem-Angriff zu erwarten und die Fähigkeiten auszustatten, um damit umzugehen, als zu versuchen, ihn um jeden Preis zu verhindern und enttäuscht zu sein, wenn es darum geht.
Sie haben gerade die Frage beantwortet: ** Folgen Sie der Sprache und den Bräuchen des Publikums, das Sie überzeugen möchten. ** Wenn Sie die Sprache nicht kennen, verwenden Sie nicht nur "Analogien" - lernen Sie die Sprache.Lesen.Studie.Hör mal zu.Sei geduldig.
@JeffE Es ist kompliziert, aber ich kann Ihnen versichern, dass es einfach unmöglich ist, geduldig zu sein.Es ist wie die Frustration, dass du keine Untreue sein willst, aber trotzdem eine Erektion bei sexy Mädchen hast.Ich denke, dass ein Anfänger (1) tatsächlich etwas sieht, was Experten auf diesem Gebiet noch nicht gesehen haben, und (2) noch keine Lehrbücher gelesen hat. Dies ist nur möglich, wenn (A) es sich um eine interdisziplinäre Forschung handelt, und(B) es basiert auf einer Analogie.Sowohl A als auch B lassen sowohl den Forscher als auch den Experten nicht einmal wissen, auf welche Felder er schauen soll, und daher nehmen beide an, dass ihr Feld das richtige Feld ist.
Um nicht anzunehmen, dass er über ihr Feld spricht und daher nicht das Gefühl hat, dass er ihr Feld beleidigt, muss sie das Feld kennen, das er zuerst lesen sollte.
_Ich kann Ihnen versichern, dass es einfach unmöglich ist, geduldig zu sein.Es ist wie die Frustration, dass du keine Untreue sein willst, aber trotzdem eine Erektion bei sexy Mädchen hast. - Und ich kann dir versichern, dass Geduld nicht nur möglich, sondern auch üblich ist.Es ist, als würde man um sexy Mädchen herum erregt und sie dann nicht angreifen.
@JeffE Sie können nur ruhig bleiben, wenn Sie darauf vertrauen, dass Ihre Liebe zu Ihrem Partner wahr oder sogar unersetzlich ist.Wenn es aus irgendeinem Grund eine Krise gibt, die Sie an Ihrer Liebe zu ihr zweifeln lässt, obwohl Sie dies tatsächlich tun, wird jede Erregung, die Sie bekommen, Ihre bestehende Angst noch verstärken.Ruhe / Geduld kann nicht erreicht werden, wenn man sich schuldig fühlt.Mit "unmöglich" meine ich die [kognitive Belastung] (https://en.wikipedia.org/wiki/Cognitive_load).Wenn wir das Lesen von Lehrbüchern zum Ziel haben, möchte ich nur untersuchen, welche Faktoren dies behindern können.


Diese Fragen und Antworten wurden automatisch aus der englischen Sprache übersetzt.Der ursprüngliche Inhalt ist auf stackexchange verfügbar. Wir danken ihm für die cc by-sa 3.0-Lizenz, unter der er vertrieben wird.
Loading...